Neurology Flashcards

1
Q

Define Guillain-Barré syndrome? Summary of how it presents?

Pathophysiology?

2 Key features in the presentation?

How is diagnosis made? 3

Management? - 3 key things

A

Guillain-Barré syndrome is an acute paralytic polyneuropathy that affects the peripheral nervous system. It causes acute, symmetrical, ascending weakness and can also cause sensory symptoms.

**^^^ If you didnt get this repeat the card!
**
It is usually triggered by an infection and is particularly associated with to Campylobacter jejuni, cytomegalovirus (CMV) and Epstein-Barr virus (EBV).Guillain-Barré is thought to occur due to a process called molecular mimicry. The B cells of the immune system create antibodies against the antigens on the triggering pathogen. These antibodies also match proteins on the peripheral neurones. They may target proteins on the myelin sheath or the nerve axon itself.

////

The characteristic features are:
Symmetrical ascending weakness
Reduced reflexes

extra: There may be peripheral loss of sensation or neuropathic pain. It may progress to the cranial nerves and cause facial weakness. Autonomic dysfunction can lead to urinary retention, ileus or heart arrhythmias.
Symptoms usually start within four weeks of the triggering infection. They begin in the feet and progress upward. Symptoms peak within 2-4 weeks. Then, there is a recovery period that can last months to years.

////
Diagnosis
Clinical diagnosis based on the Brighton criteria (weakness, loss of reflexes…)

Ix that support the diagnosis:
- Nerve conduction studies (showing reduced signal through the nerves)
- Lumbar puncture for cerebrospinal fluid (showing raised protein with a normal cell count and glucose) -> dont fully get this because its a peripheral neuropathy?!

///
Management:
VTE prophylaxis (pulmonary embolism is a leading cause of death)
**IV immunoglobulins (IVIG) first-line
**Plasmapheresis is an alternative to IVIG

Severe cases with respiratory failure may require intubation, ventilation and admission to the intensive care unit.

///

Recovery can take months to years. Patients can continue regaining function five years after the acute illness. Most patients eventually make either a full recovery or are left with minor symptoms. Some are left with significant disability. Mortality is around 5%, mainly due to respiratory or cardiovascular complications.

How well did you know this?
1
Not at all
2
3
4
5
Perfectly
2
Q

Pathophysiology of Huntington’s:

Inheritence pattern? Gene and chromosome

Type of mutation?

What can happen to successive generations in families with huntington’s? Why is this?

A

Huntington’s disease (also called Huntington’s chorea) is an autosomal dominant genetic condition that causes progressive neurological dysfunction

It is a trinucleotide repeat disorder involving a genetic mutation in the HTT gene on chromosome 4, which codes for the huntingtin (HTT) protein.

Other examples of trinucleotide repeat disorders include:

Fragile X syndrome
Spinocerebellar ataxia
Myotonic dystrophy
Friedrich ataxia

\\

Anticipation

Huntington’s chorea displays something called genetic anticipation. Anticipation is a feature of trinucleotide repeat disorders, where successive generations have more repeats in the gene, resulting in:

Earlier age of onset
Increased severity of disease

TOM TIP: Anticipation is a common concept tested in exams and worth remembering with Huntington’s.

How well did you know this?
1
Not at all
2
3
4
5
Perfectly
3
Q

Huntington’s Chorea:

How does it present?

Ix?

Management?

Prognosis and two key causes of death?

Pathophysiology is tested on another card.

A

Huntinton’s causes progressive neurological dysfunction

It typically begins with cognitive, psychiatric or mood problems, followed by the development of movement disorders:
- Chorea (involuntary, random, irregular and abnormal body movements)
- Dystonia (abnormal muscle tone, leading to abnormal postures)
- Rigidity (increased resistance to the passive movement of a joint)
- Eye movement disorders
- Dysarthria (speech difficulties)
- Dysphagia (swallowing difficulties)

Key - remember cognitive and behavoiral changes followed by movement disorder.

////

Ix- diagnosis is made by genetic testing

////
Management:
** There are currently no treatment options for slowing or stopping the progression of the disease**.

  • Tetrabenazine may be used for chorea
  • SSRIs for depression
  • physiotherapy to maintain mobility and prevent contractures, SALT to help with dysphagia and dysarthria
  • Palliative care

///
Life expectance is around 10-20 years after the onset of symptoms. As the disease progresses, patients become more frail and susceptible to illness (e.g., infections, weight loss, falls and pressure ulcers). Death is often due to aspiration pneumonia.
**Suicide ** is also a common cause of death.

How well did you know this?
1
Not at all
2
3
4
5
Perfectly
4
Q

Subarachnoid Haemorrhage:
- what is the subarachnoid space?
- What is the main cause of a SAH
- How does a subarachnoid haemorrhage present? Sx in addition to headache?

A

Subarachnoid haemorrhage involves bleeding in the subarachnoid space, where the cerebrospinal fluid is located, between the pia mater and the arachnoid membrane.

This is usually the result of a ruptured cerebral aneurysm.

Subarachnoid haemorrhage has a very high mortality (around 30%) and morbidity, making it essential not to miss.

Presentation:
The typical history is a sudden-onset occipital headache during strenuous activity, such as heavy lifting or sex. The sudden and severe onset leads to the “thunderclap headache” description. It may feel like being struck over the back of the head.

Other important features include:

Neck stiffness
Photophobia
Vomiting
Neurological symptoms (e.g., visual changes, dysphasia, focal weakness, seizures and reduced consciousness)

How well did you know this?
1
Not at all
2
3
4
5
Perfectly
5
Q

Subarachnoid Haemorrhage:
- 3 Investigations? What do they show?

  • Management? Options for repair; drug used
  • other key complication
A

Ix:
- First Line - CT head - shows hyperattenuation (white) in the subarachnoid space (basically in the fissures/sulci and the ventricles)
- Lumbar puncture: performed if the CT head appears normal (does not exclude SAH). Performed 12 hours after symptom onset to allow billirubin to accumulate in the CSF. Shows:
- Raised red cell count
- xanthochromia (yellow colour caused by billirubin)

CT angiography is used after confirming the diagnosis to locate the source of the bleeding.

///

Management:
- intubation and ventilation if conciousness is reduced
- Surgical - repair of aneurisms - can be endovascular coiling or neurosurgical clipping - both cut the anuerism off from the artery
- Nimodipine (CCB) is used to prevent vasospasm (a common complication that leads to brain ischemia)

Complications:
Hydrocephalus (increased CSF) is managed with lumbar puncture, external ventricular drain (in to a drainage bag externally) or a ventriculoperiteal shunt (into peritoneal cavity).

Remember you can view card in deck for other info on subarachnoid haem

How well did you know this?
1
Not at all
2
3
4
5
Perfectly
6
Q

KEY NEUROANATOMY FOR INTRACRANIAL HAEMORRHAGES

Which blood vessels are found in each of the four spaces between the meninges?

////

What are the layers of the BBB?

A

WARNING MADE WITH CHAT GTP BUT THINK IT IS ACCURATE, HELPFUL FOR SURE.

Epidural Space (Potential space between the skull and dura mater)
Vessels: Middle meningeal artery
The middle meningeal artery and other meningeal arteries run in the potential space between the skull and the dura mater. This space is generally “closed” unless there’s an injury.

Subdural Space (Potential space between the dura mater and arachnoid mater)
Vessels: Bridging veins
The bridging veins pass through the subdural space, connecting the superficial veins of the brain to the dural venous sinuses.

Side note for understanding: The dural venous sinuses are large venous channels located within the layers of the dura mater (the outermost layer of the meninges). These sinuses collect venous blood from the brain and cerebrospinal fluid (CSF) from the subarachnoid space and drain it into the internal jugular veins, which carry the blood back to the heart.

Subarachnoid Space (Actual space between the arachnoid mater and pia mater)
Vessels: Cerebral arteries and veins (e.g., arteries from the circle of Willis)
* The cerebral arteries (such as the anterior, middle, and posterior cerebral arteries) and veins run in the subarachnoid space, along with cerebrospinal fluid (CSF).
* A rupture of an aneurysm (e.g., a berry aneurysm) or trauma can lead to a subarachnoid hemorrhage (SAH), where blood fills the subarachnoid space, mixing with the CSF and causing irritation of the meninges (hence the thunderclap headache = meningeal irriation).

The intracerebal space (within the brain tissue itself)
Vessels: intracerebal vessels (the penetrating arteries and capillaries, and intracerebral veins)

The intracerebal arteries originate from the major cerebral arteries, which are located on the brain’s surface, in the subarachnoid space. They travel inward, passing through the pia mater (the innermost meningeal layer) and penetrating the brain tissue. The cappilaries tehy give rise to do not make contact with the meninges.

////
Key understanding - the layers of the blood brain barrier are not the meninges but are much smaller cellular surrounding the deeper vessels - endothelial cells, basement memebrane, Pericytes, astrocyte end feets and microglial immune cells. The endothelial cells that form the BBB are located within the capillaries and small arteries inside the brain parenchyma, beneath the pia mater and the subarachnoid space. These cells have tight junctions that prevent certain substances from passing freely into the brain tissue.The BBB is not directly part of the meninges but is closely associated with the vessels that pass through or are surrounded by the pia mater. The subarachnoid space contains the major cerebral arteries and veins that supply and drain blood from the brain. As these vessels penetrate into the brain tissue, they are lined with endothelial cells that are part of the BBB.

Basically the BBB is not the meniges but is cellular wrapping around the vessels both when they penetrate and more superficially when they are in the meninges.

How well did you know this?
1
Not at all
2
3
4
5
Perfectly
7
Q

Intracranial Haemorrhage:
- What are the four types?
- What vessels are found in each space?
- Whate causes the vessles to rupture?
- appaerance on CT head?
- Typical clinical presentation
- Which types of intracranial (within the skull) haemorrhages are classed as haemorrhagic strokes?

Management of intracranial bleeds

A

Intracranial haemorrhage refers to bleeding within the skull. There are four types.

Extradural haemorrhage (bleeding between the skull and dura mater)
usually caused by a rupture of the middle meningeal artery in the temporoparietal region. It can be associated with a fracture of the temporal bone. mnemonic - mixed martial arts causes fractures.

On a CT scan, they have a bi-convex shape and are limited by the cranial sutures (they do not cross the sutures, which are the points where the skull bones join together because the dura is fused to the skull at these points). LEMONS

A typical history is a young patient with a traumatic head injury and an ongoing headache. They have a period of improved neurological symptoms and consciousness, followed by a rapid decline over hours as the haematoma gets large enough to compress the intracranial contents.

Subdural haemorrhage (bleeding between the dura mater and arachnoid mater)

Caused by a rupture of the bridging veins in the outermost meningeal layer

On a CT scan, they have a crescent shape and are not limited by the cranial sutures (they can cross over the sutures - because its below the dura, the dura connects to the sutures). BANANAS

Subdural haemorrhages may occur in elderly and alcoholic patients, who have more atrophy in their brains, making the vessels more prone to rupture. They typically develop more gradually (vein not an artery)

Subarachnoid haemorrhage (bleeding in the subarachnoid space)
Bleeding in the subarachnoid space, where the cerebrospinal fluid is located, between the pia mater and the arachnoid membrane. This is usually the result of a ruptured cerebral aneurysm.

CT appearence - hyperdense (bright) areas along the sulci (grooves on the brain surface) and around the cerebral cortex. It can also be seen in the ventricular system if it spreads there.

The typical history is a sudden-onset occipital headache during strenuous activity, such as heavy lifting or sex. The sudden and severe onset leads to the “thunderclap headache” description.

Stroke - sudden onset headache (neruological symptom)

Intracerebral haemorrhage (bleeding into brain tissue)
- ME: Rupture of the intracranial vells - perforation arteries, capillaries or intracranial veins
- occours spontaneously or secondary to ischemaic stroke, tumours o aneurism rupture

  • CT appaerance - hyperdense within the brain parenchyma (looks like a white circle in a part of the brain - similar to a tumour).

It presents similarly to an ischaemic stroke with** sudden-onset focal neurological symptoms, such as limb or facial weakness, dysphasia or vision loss - it is a stroke**

stroke - sudden onset weakness, dysphagia or vision change (neurological symptom)
////

THE TWO INTRACRANIAL BLEEDS THAT ARE CLASSIFIED AS STROKES:
Intracerebral (within the brain tissue) and Subarachnoid haemorrhages.

Remeber a stroke is any sudden onset (immediate to maximum intensity) neurological symptom.

///
Management

  • Ix - CT head and coagulation screen
  • intubation and ventilation if conciousness is reduced
  • correct any clotting abnormality
  • **correct any hypertension **(big risk factor, alongside aneurism) - THIS IS KEY

Smaller bleeds may be managed conservatively with close monitoring and repeat imaging.

Surgical options for treating an extradural or subdural haematoma are:
Craniotomy (open surgery by removing a section of the skull) - expecially for the bottom two, and needed for aneurism repair)
Burr holes (small holes drilled in the skull to drain the blood) - extradural and subdurals, if small enough.

Subarachnoid can management is seperate but also involves aneurism repair using endovascular coiling or neurosurgical clipping. There is more detail and different medical management - nimodipine for preventing vasospasm.

How well did you know this?
1
Not at all
2
3
4
5
Perfectly
8
Q

The Glasgow Coma Scale (GCS) is a universal assessment tool for the level of consciousness.

  • write out all 15 steps
  • what is the most important factor progonstically?
  • what are lateralising signs?
A

It is scored based on best eye opening response, best verbal response and best motor response

Mr Carroll - dont learn the numbers learn the stages for each.

The most important prognostically is motor, if nothing else learn this. Flexing to pain or better indicates a potential to recover, extending to pain or worse indicates a poor prognosis and is such that the patient may not be accepted by a neurosurgeon for treatment.

Best Motor Response
6. Obeys commands (raise you arms above your head)
5. Localises to pain (bats hands away)
4. Flexes to pain (reachs up to face but not all the way)
(3. Spastic flexion/’withdrawal’) AVOID USING THIS HEADING - harder to rememebr
2. Extends to pain - (straightens arms out in repsosne to pain)
1. None
LEARN THIS ^^^

Best Verbal Response
5. Orientated in time, place, and person
4. Confused
3. Inappropriate words
2. Incomprehensible sounds
1. None
Best Eye Opening Response
4. Eyes open spontaneously
3. Eyes open to speech
2. Eyes open to pain
1. None

The best site for applying a pain stimulus is a point immediately below the ear between mastoid process posteriorly and mandible anteriorly, applying the stimulus bilaterally, equally, and ultimately as hard as one can.

///

A lateralising sign reflects a problem with one hemisphere versus the other:
- inattention to one side
- gaze paresis (unable to look to one side)
- slower or asymetrical motor response versus the other.

How well did you know this?
1
Not at all
2
3
4
5
Perfectly
9
Q
  • What are the two types of Cerebrovascular accidents
  • 4 Causes of Ischemia?
  • Definition of a TIA
  • presentation of a stroke?
A

within 1/2 hours. A stroke is a cerebrovascular event that is caused by abnormal perfusion of cerebral tissue. Cerebrovascular accidents are either:

  • Ischaemia or infarction of the brain tissue secondary to a disrupted blood supply (ischaemic stroke)
  • Intracranial haemorrhage, with bleeding in or around the brain (haemorrhagic stroke)

A sudden onset of any neurological symptoms suggests a vascular cause (e.g., stroke). Stroke symptoms are typically asymmetrical.

The blood supply to the brain may be disrupted by:

A thrombus or embolus
Dissection of the carotid artery
Atherosclerosis

Transient ischaemic attack (TIA) involves temporary neurological dysfunction (lasting less than 24 hours) caused by ischaemia but without infarction. Symptoms have a rapid onset and often resolve before the patient is seen. TIAs may precede a stroke. Crescendo TIAs are two or more TIAs within a week and indicate a high risk of stroke.

Common symptoms are sudden-onset, typically asymetrical:

Limb weakness
Facial weakness
Dysphasia (speech disturbance)
Visual field defects
Sensory loss
Ataxia and vertigo (posterior circulation infarction)

FAST mnemonic - sudden onset (fast) fascial weakness, arm (limb) weakness, dysphasia (speech disturnace). But also remember the other three

How well did you know this?
1
Not at all
2
3
4
5
Perfectly
10
Q

What is the anterioir and the posterior circulation and which vessels make up each?

What do the three cerebral arteries each supply?

A

The network of blood vessels that supply the brain are joined by the circle of Willis.
Many blood vessels supply different areas of the brain. These are broadly divided into the anterior and posterior circulation.

Anterior circulation: a network of blood vessels arising from the carotid arteries.

Posterior circulation: a network of blood vessels arising from the vertebrobasilar arteries.

Carotid system: the common carotid artery gives off the internal and external carotid arteries. The internal carotid artery is vital to the circle of Willis. It gives off the anterior and middle cerebral arteries as well as the posterior communicating artery.

Vertebrobasilar system: the two vertebral arteries join to form the basilar artery. Multiple branches arise from this artery. The basilar artery gives off the posterior cerebral arteries and joins anterior circulation via the posterior communicating artery.

Individual blood vessels correspond to areas of the brain that have specific motor, sensory, and/or high cortical functioning (e.g. speech).

///

Cerebral arteries

There are three major cerebral vessels, which are connected by communicating arteries at the circle of Willis.

Anterior cerebral artery: supplies part of the frontal and parietal lobe. Part of the anterior circulation of the brain
Middle cerebral artery: supplies a large proportion of the lateral surface of each brain hemisphere including the internal capsule and basal ganglia. Most common site of infarction. Forms part of the anterior circulation of the brain
Posterior cerebral artery: supplies the occipital lobe and inferior proportion of the temporal lobe as well as some deep structures (e.g. thalamus). Forms part of the posterior circulation of the brain
Occlusion of these arteries leads to a characteristic set of clinical features corresponding to the motor and/or sensory functions they help control.

How well did you know this?
1
Not at all
2
3
4
5
Perfectly
11
Q

A stroke presents with sudden, focal neurological deficit that reflects the area of brain devoid of blood flow. What how do anterior and posteroir infarctions presents?

How does the Bamford/oxford classification tie into this - what are the requirements for the 4 types of stroke?

A

Anterior ischaemic stroke:

Patients with anterior ischaemic strokes (i.e. TACS, PACS, LACS) develop a constellation of features dependent on the extent and location of the infarct.

1.Unilateral weakness and/or sensory deficit: face and/or arms and/or legs
2.Homonymous hemianopia: visual field loss on the same side of both eyes (make sense when you look up the tracts)
3.Higher cerebral dysfunction: dysphasia (language), visuospatial dysfunction (e.g. neglect, agnosia (recogniton))

Classically, an isolated infarction of the anterior cerebral artery leads to contralateral leg weakness only.

Bamford:
- Total anterioir circulation stroke - affecting both the ACA and MCA has all 3 of the above
- Partial anteroir circulation stroke - affects either the ACA or the MCA and causes 2/3 of the above criteria
- Lucunar stroke - is subcortical and affects the deep perorating arteries supplying deep areas of the brain (for example, a stroke in the thalamus, the basal ganglia or pons) is called a lacunar stroke.

Lacunar syndromes: (1) pure motor hemiparesis. (2) pure sensory. (3) ataxic hemiparesis. (4) Dysarthria-clumsy hand syndrome. (5) sensorimotor

///

Posterior ischaemic stroke

The posterior circulation is composed of the vertebrobasilar arterial system. This supplies the brainstem, cerebellum and occipital cortex. Therefore, posterior strokes can affect balance, vision, and cranial nerves. POCS account for 20-25% of ischaemic strokes

Dizziness
Diplopia
Dysarthria & Dysphagia
Ataxia
Visual Field defects
Brainstem syndromes: for example basilar artery leads to locked in syndrome, Posteroir inferioir cerebellar (not PCA) occlusion affects the medulla - Nystagmus, vertigo, diplopia, horners sydnrome, facial sensory loss, dysarthria

Bamford:
Posteroir circulation circulation strokes (1) Brainstem or cerebellar syndrome. (2) loss of consciousness. (3) Isolated homonymous hemianopia

How well did you know this?
1
Not at all
2
3
4
5
Perfectly
12
Q

Initial management of TIA, including Ix of choice - 4 THINGS

Initial management of Stroke including timelines for each option (3 main things)

A

Symptoms should have completely resolved within 24 hours of onset. Initial management involves:

The principle management for TIA is 300 mg of aspirin that is usually continued for two weeks.

This is followed by treatment with 75 mg of clopidogrel as long-term vascular prevention.

Referral for specialist assessment within 24 hours (within 7 days if more than 7 days since the episode) - for bloods ECG and imaging (below carotid and MRI)

Diffusion-weighted MRI scan is the imaging investigation of choice (i think it shows no dammage because there is no infarction).

Carotid dopplers are also performed

////

Management of Stroke
Initial management involves:

Exclude hypoglycaemia
Immediate CT brain to exclude haemorrhage
Aspirin 300mg daily for two weeks (started after haemorrhage is excluded with a CT)
Admission to a specialist stroke centre

Thrombolysis with alteplase is considered once haemorrhage is excluded (after the CT scan). Alteplase is a tissue plasminogen activator that rapidly breaks down clots. It may be given within 4.5 hours of the symptom onset, based on local protocols and by an appropriately trained team. Patients need close monitoring for complications, particularly intracranial or systemic haemorrhage, with access to immediate imaging if bleeding is suspected.

Thrombectomy is considered in patients with a confirmed blockage of the proximal anterior circulation or proximal posterior circulation. It may be considered within 24 hours of the symptom onset and alongside IV thrombolysis.

In patients with an ischaemic stroke, lowering the blood pressure can worsen the ischaemia. High blood pressure treatment is only indicated in hypertensive emergency or to reduce the risks when giving intravenous thrombolysis. Blood pressure is aggressively treated in patients with a haemorrhagic stroke.

How well did you know this?
1
Not at all
2
3
4
5
Perfectly
13
Q

Ongoing management of stroke, after the immediate management:

How are patients with TIA or Stroke assessed for underlying causes?

How are each of these causes managed?

Secondary prevention of stroke? - 4 things

A

Patients with a TIA or stroke are investigated for carotid artery stenosis and atrial fibrillation with:

Carotid imaging (e.g., carotid ultrasound, or CT or MRI angiogram)
ECG or ambulatory ECG monitoring

Anticoagulation is initiated for atrial fibrillation (after excluding haemorrhage and finishing two weeks of aspirin).

Surgical interventions are considered where there is significant carotid artery stenosis. The options are:

Carotid endarterectomy (recommended in the NICE guidelines)
Angioplasty and stenting

TOM TIP: The top risk factors to remember are atrial fibrillation and carotid artery stenosis. All patients with a TIA or stroke will have carotid imaging and ECGs to identify these.

////

Secondary Prevention

Clopidogrel 75mg once daily (alternatively aspirin plus dipyridamole)
Atorvastatin 20-80mg (not started immediately – usually delayed at least 48 hours)
Blood pressure and diabetes control
Addressing modifiable risk factors (e.g., smoking, obesity and exercise)

Aspirin is aleady initiated in the immediate management and i think continues

////
*Not testing myself but just be aware
*
Stroke patients require a period of adjustment and rehabilitation involving a multi-disciplinary team of:

Stroke physicians
Nurses
Speech and language (SALT) to assess swallowing
Dieticians in those at risk of malnutrition
Physiotherapy
Occupational therapy
Social services
Optometry and ophthalmology
Psychology
Orthotics

How well did you know this?
1
Not at all
2
3
4
5
Perfectly
14
Q

Atrial fibrillation:
Uncontrolled and unorganised activity in the atria leads to blood stagnating in the left atrium, particularly in the left atrial appendage. Eventually, this stagnated blood leads to a thrombus (clot). This thrombus then mobilises (becomes an embolus) and travels from the left atrium to the left ventricle, into the aorta and up in the carotid arteries to the brain. It can then lodge in a cerebral artery and cause an ischaemic stroke.

Without anticoagulation, patients with atrial fibrillation have around a 5% risk of stroke each year, depending on individual factors.With anticoagulation, patients with atrial fibrillation have around a 1-2% risk of stroke each year, depending on individual factors. Anticoagulation treatment carries around a 2.5-8% risk of serious bleeding each year, depending on individual factors.

QUESTION:

What two scores are used to calculate clot vs haemorrhage risk in patients with atrial fibrillation? Name the parts to each score and interpret the overall results…

Extra point - What is the immediate management for head injuries for patients that are on anti-coagulation?

A

CHA2DS2-VASc

CHA2DS2-VASc is a tool for assessing whether a patient with atrial fibrillation should start anticoagulation. It is a list of risk factors that increase the likelihood of a stroke. The higher the score, the higher the risk of developing a stroke or TIA.

CHA2DS2-VASc is a mnemonic for the factors that score a point:

C – Congestive heart failure
H – Hypertension
A2 – Age above 75 (scores 2)
D – Diabetes
S2 – Stroke or TIA previously (scores 2)
V – Vascular disease
A – Age 65 – 74
S – Sex (female)

NICE (2021) recommends, based on the CHA2DS2-VASc score:

0 – no anticoagulation
1 – consider anticoagulation in men (women automatically score 1)
2 or more – offer anticoagulation

Bleeding Risk

The NICE guidelines recommend using the ORBIT score for assessing the risk of major bleeding in patients with atrial fibrillation taking anticoagulation. The easiest way to calculate the ORBIT score is using an online calculator. The “ORBIT” mnemonic can be used to remember the 5 factors:

O – Older age (age 75 or above)
R – Renal impairment (GFR less than 60)
B – Bleeding previously (history of gastrointestinal or intracranial bleeding)
I – Iron (low haemoglobin or haematocrit)
T – Taking antiplatelet medication

For most patients with atrial fibrillation, the risk of stroke with no anticoagulation will outweigh the risk of bleeding on anticoagulation.

The NICE guidelines (2021) recommend for anticoagulation:

Direct-acting oral anticoagulants (DOACs) first-line
Warfarin second-line, if DOACs are contraindicated

TOM TIP: Every patient with a head injury whilst taking anticoagulation should have a CT head to assess for an intracranial bleed, according to the NICE guidelines on head injuries (updated 2019). Being asked to review a patient after a fall is very common, and it helps to remember that a head injury plus anticoagulation automatically qualifies them for a CT scan. Inform patients starting on anticoagulation that they will need medical attention (A&E) in the event of a head injury for this reason.

How well did you know this?
1
Not at all
2
3
4
5
Perfectly
15
Q

Ataxia vs Dyskinesia

A

THIS IS FROM CHAT GTP

Ataxia

** * Definition: Ataxia is a lack of coordination and control over voluntary movements, usually caused by damage to the cerebellum, the part of the brain responsible for motor coordination and balance.**

*	Characteristics:
*	Unsteady Gait: Difficulty walking and maintaining balance (ataxic gait).
*	Poor Coordination: Difficulty performing fine motor tasks like buttoning clothes or writing.
*	Impaired Speech: Slurred or slow speech (dysarthria).
*	Inconsistent Movements: Movements can be jerky and poorly coordinated, leading to tremors during voluntary actions.
*	Eye Movement Problems: Difficulty controlling eye movements (nystagmus).
*	Causes: Cerebellar damage (stroke, tumors, trauma), hereditary conditions (Friedreich’s ataxia, spinocerebellar ataxia), toxins, alcohol abuse, vitamin deficiencies.
*	Main Issue: Lack of coordination in movement.

Dyskineasia  **	Definition: Dyskinesia refers to involuntary, abnormal movements that occur due to dysfunction in the basal ganglia, the part of the brain involved in regulating voluntary movements.**

*	Characteristics:
*	Involuntary Movements: Movements that occur without conscious control, such as writhing, twisting, or jerking.
*	Types of Movements:
*	Chorea: Rapid, unpredictable, dance-like movements.
*	Athetosis: Slow, writhing movements.
*	Tremors: Rhythmic, shaking movements, typically seen in conditions like Parkinson’s disease.
*	Commonly Seen in Parkinson’s Disease: Dyskinesia often results from long-term use of medications like levodopa used to treat Parkinson’s disease. It can appear as writhing or jerking movements.
*	Tardive Dyskinesia: Involuntary movements, especially in the face or jaw, resulting from prolonged use of certain antipsychotic medications.
*	Causes: Medications (e.g., levodopa, antipsychotics), Parkinson’s disease, Huntington’s disease, brain injuries, and other conditions affecting the basal ganglia.
*	Main Issue: Involuntary movements that are excessive or erratic.
How well did you know this?
1
Not at all
2
3
4
5
Perfectly
16
Q

Your Supervisor said there are 4 key neurological emergencies

A
  1. Stroke
  2. Sudden onset headache
  3. seisure
  4. rapidly accending weakness
How well did you know this?
1
Not at all
2
3
4
5
Perfectly
17
Q

Types of Seizures seen in adults (5) and children (3)

Give a description of each seizsure type

A

The types generally seen in adults include:

Generalised tonic-clonic seizures
Partial seizures (or focal seizures)
Myoclonic seizures
Tonic seizures
Atonic seizures

The types more common in children include:

Absence seizures
Infantile spasms
Febrile convulsions

Generalised tonic-clonic seizures involve tonic (muscle tensing) and clonic (muscle jerking) movements associated with a complete loss of consciousness. Typically, the tonic phase comes before the clonic phase. They are also called grand mal seizures. Before the seizure, patients might experience aura, an abnormal sensation that gives a warning that a seizure will occur. There may be tongue biting, incontinence, groaning and irregular breathing. After the seizure, there is a prolonged post-ictal period, where the person is confused, tired, and irritable or low.

Partial seizures (or focal seizures) occur in an isolated brain area, often in the temporal lobes. They affect hearing, speech, memory and emotions. Patients remain awake during partial seizures. They remain aware during simple partial seizures but lose awareness during complex partial seizures. There are various symptoms associated with partial seizures, depending on the location of the abnormal electrical activity:

Déjà vu
Strange smells, tastes, sight or sound sensations
Unusual emotions
Abnormal behaviours

Myoclonic seizures present with sudden, brief muscle contractions, like an abrupt jump or jolt. They remain awake. Myoclonic seizures can occur as part of juvenile myoclonic epilepsy in children.

Tonic seizures involve a sudden onset of increased muscle tone, where the entire body stiffens. This results in a fall if the patient is standing, usually backwards. They last only a few seconds, or at most a few minutes.

Atonic seizures (causing “drop attacks”) involve a sudden loss of muscle tone, often resulting in a fall. They last only briefly, and patients are usually aware during the episodes. They often begin in childhood. They may be indicative of Lennox-Gastaut syndrome.

Absence seizures are usually seen in children. The patient becomes blank, stares into space, and then abruptly returns to normal. During the episode, they are unaware of their surroundings and do not respond. These typically last 10 to 20 seconds. Most patients stop having absence seizures as they get older.

Infantile spasms are also known as West syndrome. It is a rare (1 in 4,000) disorder starting at around six months of age. It presents with clusters of full-body spasms. Hypsarrhythmia is the characteristic EEG finding. It is associated with developmental regression and has a poor prognosis. Treatment is with ACTH and vigabatrin.

Febrile convulsions are tonic-clonic seizures that occur in children during a high fever. They are not caused by epilepsy or other pathology (e.g., meningitis or tumours). Febrile convulsions occur in children aged between 6 months and 5 years. Febrile convulsions do not usually cause any lasting damage. One in three will have another febrile convulsion. They slightly increase the risk of developing epilepsy.

How well did you know this?
1
Not at all
2
3
4
5
Perfectly
18
Q

Epilepsy:

Ix?

Long-term Management?

A

An electroencephalogram (EEG) shows typical patterns in different forms of epilepsy and supports the diagnosis.

MRI brain is used to diagnose structural pathology (e.g., tumours).

Additional investigations can be considered to exclude associated pathology:
ECG - cardiac syncope
Serum electrolytes, including sodium, potassium, calcium and magnesium
Blood glucose - rule out hypoglycaemia and diabetes
Blood cultures, urine cultures and lumbar puncture where sepsis, encephalitis or meningitis is suspected

Management
- inform the DVLA until seizure free for a year
- Women - Lamotrigine or Levetiracetam for all seizure types, except ethosuximate used for absent seizsures
- Zero to finals still says men - sodium valproate first line for generalise and tonic seizsures, However, i think this is outdated because of the sodium valproate pregnancy prevention programme…

How well did you know this?
1
Not at all
2
3
4
5
Perfectly
19
Q

What are the side effects of sodium valproate - name 3

Who can it not be prescribed to?

A

Sodium valproate works by increasing the activity of gamma-aminobutyric acid (GABA), which has a calming effect on the brain. Notable side effects include:

**Teratogenic (harmful in pregnancy) - causes neural tube defects and developmental delay
**Liver damage and hepatitis
Hair loss
Tremor
Reduce fertility

The new restrictions mean no one under the age of 55 will be newly prescribed sodium valproate unless two specialists agree there is no other effective or tolerated treatment, or unless there are “compelling reasons that the reproductive risks do not apply”. It used to apply to women who had to be on a form of contraception and sign a form acknowledging the risks of valproate.

How well did you know this?
1
Not at all
2
3
4
5
Perfectly
20
Q

Define Status Epilepticus?

What is the management of status?
- First, Second and Third line
- 3 formulations for first line?

A

Status epilepticus is a medical emergency defined as either:

A seizure lasting more than 5 minutes
Multiple seizures without regaining consciousness in the interim

////

It is managed with an ABCDE approach invovling securing the airway, giving oxygen and ensuring IV access, and checking BMs

Medical treatment involves:

A benzodiazepine first-line, repeated after 5-10 minutes if the seizure continues
Second-line options (after two doses of benzodiazepine) are IV levetiracetam, phenytoin or sodium valproate
Third-line options are phenobarbital or general anaesthesia

Options for benzodiazepines are:

Buccal midazolam (10mg) - M for mouth
Rectal diazepam (10mg) - D for derrier
Intravenous lorazepam (4mg) - l looks like an I for IV

How well did you know this?
1
Not at all
2
3
4
5
Perfectly
21
Q

Presentation of Syncope vs Seizure

A

Syncope vs seizure

Prolonged upright position before the event, lightheadedness, clammy, blurred vision before event
VS Epilepsy aura - smells, tastes, deja vu before the event

Reduced tone during the episode (although there may be some convulsion activity in syncope)
VS Tonic clonic activity, abnormal limb movmeent, head turning and may have tongue biting

Return of consciousness shortly after falling - seconds to a minute as blood flow returns to the brain
VS lasts more than five minutes

No prolonged post-ictal period - they may feel a bit groggy
VS postictal seizures patients have prolonged confusion, drowsiness, irritability and disorientation

There may be incontinence with both seizures and syncopal episodes.

How well did you know this?
1
Not at all
2
3
4
5
Perfectly
22
Q

Pathophysiology of parkinsons

Classic Triad of Parkinsonism

Sx of Parkinson’s - key features and non-motor symptoms

A

Parkinson’s disease is a condition where there is a progressive reduction in dopamine in the basal ganglia, leading to disorders of movement. The symptoms are characteristically asymmetrical, with one side of the body affected more. The typical patient is an older man, around 70 years old, with a gradual onset of symptoms.

There is a classic triad of features in Parkinson’s disease:

Resting tremor (a tremor that is worse at rest)
Rigidity (resisting passive movement - muscle resist stretch/tight)
Bradykinesia (slowness of movement)

How to remeber it is resting tremor - the basal ganglia are all about controlling voluntary movements and when their function is affected you get involuntary movements (disinhibition). They are also responsible for co-ordinating habitual movements like walking, which slow down as their function is lost.

Background - not tested as such
The basal ganglia are a group of structures situated near the centre of the brain. They are responsible for coordinating habitual movements such as walking, controlling voluntary movements and learning specific movement patterns.
Dopamine plays an essential role in the basal ganglia function. Patients with Parkinson’s disease have a slow but progressive drop in dopamine production.

///

Features:

Resting “pill-rolling” tremor - improves with movement

Cogwheel rigity - tremor on top of rigidity

Bradykinesia (slower, smaller movements) presents as:
- hypomimia (reduced facial expression)
- Gait: loss of arm swing is an early sign, shuffling gait, difficulty initiating movement and turning around.
- micrographia (smaller handwriting)

  • hypomimia
  • slowness of movement (bradykinesia), fatiguing (slower with time)
  • tremor is a rest tremor (when the hand is relaxed) - 70%
  • shuffling gait

^^^ i like this way to remeber it with 3 things within the bradykineasia

Also presents with other non-motor features:
- sleep disturbance and insomnia
- cognitive impairement and memory problems
- postural instability (Risk of falls)
- anosmia - loss of sense of smell

How well did you know this?
1
Not at all
2
3
4
5
Perfectly
23
Q

Parkinson’s Tremor vs Benign Essential tremor

A

Parkinson’s
- Asymetrical
- worse at rest
- Other parkinson’s features
- worse with alcohol
- slower 4-6Hz

Benign Essential tremor:
- symmetrical (although not always i think)
- worse with movement (cant drink from a cup)
- isolated tremor
- improves with alcohol - Jane loves a drink
- Faster 6-12Hz, also commonly not just hands (Jane’s voice and head move)

How well did you know this?
1
Not at all
2
3
4
5
Perfectly
24
Q

Parkinson’s Disease:
- How is diagnosis made

Management:
- 4 treatment options
- synthetic dopamine is always combined with?
- an example of each
- basic mechanism of action for each?
- Side effect of Levodopa, Rx?

A

Clinical diagnosis based on signs (examination) and symptoms (history)

DaT (Dopamine Transmission) scan shows loss of doperminergic neurones in the basal ganaglia when there is doubt surroudning the diagnosis. (Comma shaped (normal) -> full stop with (loss in parkinsons)

///
Management:

No treatment slows progression, they are about symptomatic control. Patients may describe themselves as “on” when the medications are acting, and they are moving freely, and “off” when the medications wear out, they are experiencing symptoms and their next dose is due.

The treatment options are:

Levodopa (combined with peripheral decarboxylase inhibitors, which stop it being metabolised in the body before reaching the brain)
COMT inhibitors
Dopamine agonists
Monoamine oxidase-B inhibitors

//
Synthetic Dopamine - Levodopa: Co-beneldopa (levodopa and benserazide)

Side effect is dyskinesia (excessive motor activity):
- dystonia (abnormal movements)
- chorea (involvuntary moevements)
- athetosis (involuntary twisting or writhing movements)

Rx with amantadine (glutamate antagonist)

This is the most effective treatment so saved until it is needed

//
COMT Inhibitors - Entacapone (C-apone for Comt)

COMT metaolises levodopa in the body (mainly) and the brian. It can be needs to be taken with Levodopa and it extends its half life.

//
Dopamine Agonists - Bromocryptine
stimulate dopamine receptors but are different structure to dopamine - less effective
Me - Dopamine inhibitors pituitary - bromocryptine used in hyperporlactinaemia -
//
MOA-B inhibitors- Selegiline (selective for dopamine breakdown)
mononamine (dopamine) oxidase (breakdown) inhibitors - inhibitors dopamine breakdown in the brain. Selective for dopamine over other monoamines like glutamate, seritonin….

How well did you know this?
1
Not at all
2
3
4
5
Perfectly
25
Q

What are Parkinson’s plus syndromes?

Key additional features 3/4

A

Parkinson’s plus syndromes are conditions that have the same classical triad of parkinson’s symptoms but with further symptoms:

Multiple Systems atrophy - multiple areas of the brain degenerate, including the basal ganglia (hence parkinsonism) but also there is widespread autonomic dysfunction (postural hypotension, abnormal sweating and sexual dysfuntion)

Demmentia with Lewy bodies - Parkinsonism with progressive conitive declcine,** visual hallucinations**, delusions, REM sleep disorders

Progressive supranuclear palsy - in the name affects eye movement (supranuclear area int he brainstem control eye movement)

Corticobasal degeneration

Id say lower yield but kist learn

MS- autonomic dysfuntion
demmentia with LB - visual hallucinations and congitive impairement
PSP - eye movement disorders

How well did you know this?
1
Not at all
2
3
4
5
Perfectly
26
Q

What is benign essential tremor?

Features?

Management? -2

A

Benign essential tremor is a relatively common condition associated with older age. It is characterised by a fine tremor affecting all the voluntary muscles. It affects voluntary movements - It is most notable in the hands but can affect other areas, for example, causing a head tremor, jaw tremor and vocal tremor.

Features

Fine tremor (6-12 Hz)
Symmetrical
**More prominent with voluntary movement - this is key, Jane can’t hold a cup
**Worse when tired, stressed or after caffeine
Improved by alcohol - Jane drinks because of her tremor
Absent during sleep

Diagnosis is clinical.

Rx:
There is no definitive treatment for benign essential tremor. Medications that may improve symptoms are:

Propranolol (a non-selective beta blocker)
Primidone (a barbiturate anti-epileptic medication)

How well did you know this?
1
Not at all
2
3
4
5
Perfectly
27
Q

Myasthenia Gravis:

  • what is myasthenia gravis?
  • associated condition
  • pathophysiology
  • presentation?
A

Myasthenia means affecting the Neuromuscular juntion. Myasthenia gravis is an autoimmune condition affecting the neuromuscular junction that gets progressively worse with activity and improves with rest.

Myasthenia gravis affects men and women at different ages, typically affecting women under 40 and men over 60.

There is a strong link with thymomas (thymus gland tumours). 10-20% of patients with myasthenia gravis have a thymoma. 30% of patients with a thymoma develop myasthenia gravis.

Pathophysiology:
- Acetylcholine receptor Antibodies bind to the post synaptic acetyl chiline receptors, blocking them from recieving stimulation from Ach. The more the receptors are used in muscle contraction, the more they become blocked causing progressive weakness with activity.

Presentation:
Can be mild or life-threaningly severe. Most commonly affects the proximal muscles of the limsb and the small muscles of the head and neck :
- Difficulty climbing stairs, standing from a seat or raising their hands above their head
- Extraocular muscle weakness, causing double vision (diplopia)
- Eyelid weakness, causing drooping of the eyelids (ptosis)
- Weakness in facial movements
- Difficulty with swallowing
- Fatigue in the jaw when chewing
- Slurred speech

On examination - repeated blinking exercerbates ptosis, prolongued upward gazing exercerbated diplopia, and repated arm abduction will result in weakness

Different from Guillaine Barre - ascending parlytic polyneuropathy - emergency with rapidly accending weakness, starting in the legs, caused by molecular mimicry from campy antibodies (after gastro) attacking motor neurones/myeline sheath. Myasthenia gravis is a chronic condition. Just putting this in because you used to get confused but i think startin to clear this up now.

How well did you know this?
1
Not at all
2
3
4
5
Perfectly
28
Q

Myathenia Gravis Cont…

  • Ix
  • Treatment options?
  • What is a myathenic crisis and how is it managed?
A

Ix:

Antibody tests look for:

AChR antibodies (around 85%) - just be aware of the others…
MuSK antibodies (less than 10%)
LRP4 antibodies (less than 5%)

CT or MRI of the thymus gland to look for thymoma

Gold standard - Endophonium test where there is doubt - neostigmine (AchEstare inhibitor relieves the weakness)

NOT CONDUCTION STUDIES - THAT IS GUILLAIN BARRE, demylination, in myathenia the nerve conduction is fine, its the NMJ!!!

///

Treatment options:
Treatment options include:

First line - Pyridostigmine is a cholinesterase inhibitor that prolongs the action of acetylcholine and improves symptoms
Immunosuppression (e.g., prednisolone or azathioprine) suppresses the production of antibodies
Thymectomy can improve symptoms, even in patients without a thymoma
Rituximab (a monoclonal antibody against B cells) is considered where other treatments fail

////
Myasthenic Crisis

Myasthenic crisis is a potentially life-threatening complication of myasthenia gravis. It causes an acute worsening of symptoms, often triggered by another illness, such as a respiratory tract infection. Respiratory muscle weakness can lead to respiratory failure. Patients may require non-invasive ventilation or mechanical ventilation.

Treatment is with IV immunoglobulins and plasmapheresis. - SAME AS GB

OMG THIS IS WHERE THE CONFUSION COMES FROM WITH HUILLAINE BARRER, A MYATHENIC CRISIS IS A BIT LIKE GUILLANE BARRE WHERE YOU GET RESPIRATORY INVOVLEMENT AND BOTH ARE MANAGED THE SAME WAY. BOTH ARE CAUSED BY ANTIBODIES BUT MYASTHENIA TO THE ACHR IN THE NMJ, GB TO THE MYELIN SHEATH AFFECTING NERVE CONDUCTION

remember can browse in deck to look at the presentation and patho

How well did you know this?
1
Not at all
2
3
4
5
Perfectly
29
Q

A brief summary on lambert-eaton myathenic syndrome

A

Lambert-Eaton myasthenic syndrome is an autoimmune condition affecting the neuromuscular junction, similar to myasthenia gravis. The symptoms tend less pronounced than myasthenia gravis.In most cases, it is a paraneoplastic syndrome occurring alongside small-cell lung cancer (SCLC). It can also occur as a primary autoimmune disorder without the presence of SCLC.

Symptoms are similar with proximal muscle weakness but, in contrast to with MG, symptoms improve with activity and are worse at rest.

Excluding underlying malignancy (e.g., small-cell lung cancer) is essential.

I wouldnt learn much more than this, too much to memorise but reacon could save your skin in an exam one day

How well did you know this?
1
Not at all
2
3
4
5
Perfectly
30
Q

headahces, GC arterities and migranes, then facial nerve and Herpes Zoster.

A
How well did you know this?
1
Not at all
2
3
4
5
Perfectly
31
Q

Card 1 Multiple Sclerosis:

Pathophysiology of Multiple Sclerosis?

Which part of the nervous system is affected?

Typically age of onset?

How long does it take for the onset of symptoms in MS?

Describe the 4 Disease patterns?

A

Multiple sclerosis (MS) is a chronic and progressive autoimmune condition involving demyelination in the central nervous system. The immune system attacks the myelin sheath of the myelinated neurones.

Myelin covers the axons of neurones and helps electrical impulses travel faster. Myelin is provided by cells that wrap themselves around the axons:

Oligodendrocytes in the central nervous system
Schwann cells in the peripheral nervous system

  • ME - note the 1st and 2nd cranial nerves are mylenated by oligodendrocytes (the rest are schwann cells) hence why optic neuritis occours in MS.

Multiple sclerosis affects the central nervous system (the oligodendrocytes). Inflammation and immune cell infiltration cause damage to the myelin, affecting the electrical signals moving along the neurones.

When a patient presents with symptoms of an MS attack (e.g., an episode of optic neuritis), there are often other demyelinating lesions throughout the central nervous system, most of which are not causing symptoms.

In early disease, re-myelination can occur, and the symptoms can resolve. In the later stages of the disease, re-myelination is incomplete, and the symptoms gradually become more permanent.

A characteristic feature of MS is that lesions vary in location, meaning that the affected sites and symptoms change over time. The lesions are described as “disseminated in time and space”. ME - THINK OF THE NAME Multiple sclerosis - hardening in multiple different places

///

Multiple sclerosis typically presents in young adults (under 50 years) and is more common in women.

///

Symptoms usually progress over more than 24 hours. Symptoms tend to last days to weeks at the first presentation and then improve. There are many ways MS can present, depending on the location of the lesions.

///

Disease patterns - a wide spectrum (some have mild relapsing and remitting symptoms, others have progressive disease).

Clinically isolated syndrome - first episode of demylination and neurological symptoms. Patients with clinically isolated syndrome may never have another episode or may go on to develop MS. Lesions on an MRI scan suggest they are more likely to progress to MS.

Relapsing-remitting MS is the most common pattern when first diagnosed. It is characterised by episodes of disease and neurological symptoms followed by recovery. The symptoms tend to occur in different areas with each episode.

Secondary Progressive - MS where there was orinally relapsing-remitting disease byt not there is progressive worsening of symptoms without complete remissions

Primary progressive - worsesning disease and neurological symptoms from the point of diagnosis without relapses and remissions.

How well did you know this?
1
Not at all
2
3
4
5
Perfectly
32
Q

Card 2 - Multiple Sclerosis Symptoms:
- 4 broad catergories of MS symtoms for learning
- what are the 4 key features of optic neuritis?
- how is optic neuritis managed?
- Symptoms due to abnormalities of eye movements?
- list the focal neurological symptoms of MS, what two subcategories are there?
- What causes Lhermitte’s sign
- what is the name of the pathological process in the spine responsible for these focal symptoms

A

ME - MS can basically cause any neurological symptom so if not sure, it could be MS. But the 4 key categories of sympotms for learning (you’ve made these up btw) are:

  1. optic neuritis (the most common presentation)
  2. eye movement abnormalities - nystagmus and diplopia
  3. focal neurological symptoms - key
  4. ataxia

//
Optic neuritis is the most common presentation of multiple sclerosis. It involves demyelination of the optic nerve and presents with unilateral reduced vision, developing over hours to days.

Key features are:

Central scotoma (an enlarged central blind spot)
Pain with eye movement
Impaired colour vision
Relative afferent pupillary defect

A relative afferent pupillary defect is where the pupil in the affected eye constricts more when shining a light in the contralateral eye than when shining it in the affected eye. When testing the direct pupillary reflex, there is a reduced pupil response to shining light in the eye affected by optic neuritis. However, the affected eye has a normal pupil response when testing the consensual pupillary reflex. So the afferent pupillary reflex is defective basically.

Optic neuritis also occours in other rheumatological and infectious diseases.

Optic neuritis is treated with high-dose steroids. Changes on an MRI scan help to predict which patients will go on to develop MS.

//

Eye movement abnormalities:
- Lesions affecting the oculomotor (CN III), trochlear (CN IV) or abducens (CN VI) can cause double vision (diplopia) and nystagmus.
- I think alot of the issues of eye movement are due to the nerve fibres of the medial longitudinal fasciulus that connects the cranial nerve nuclei.
- Diplopia is caused by impaired abduction of one eye

NOTE - Alot of this stuff i skipped because i think its relative low yield, i would just know that in addition to optic neuritis there can also be eye movement abnormalities

//

Multiple sclerosis may present with focal weakness, for example:

Incontinence
Horner syndrome
Facial nerve palsy
Limb paralysis

Multiple sclerosis may present with focal sensory symptoms, for example:

Trigeminal neuralgia/neuropathic pain
Numbness
Paraesthesia (pins and needles)
Lhermitte’s sign

Lhermitte’s sign describes an electric shock sensation that travels down the spine and into the limbs when flexing the neck. It indicates disease in the cervical spinal cord in the dorsal column. It is caused by stretching the demyelinated dorsal column.

Transverse myelitis refers to a site of inflammation in the spinal cord, which results in sensory and motor symptoms depending on the location of the lesion.

How well did you know this?
1
Not at all
2
3
4
5
Perfectly
33
Q

Define Ataxia?

What are the two types

A

Ataxia is a problem with coordinated movement. It can be sensory or cerebellar.

Sensory ataxia is due to loss of proprioception, which is the ability to sense the position of the joint (e.g., is the joint flexed or extended). This results in a positive Romberg’s test (they lose balance when standing with their eyes closed) and can cause pseudoathetosis (involuntary writhing movements). A lesion in the dorsal columns of the spine can cause sensory ataxia.

Cerebellar ataxia results from problems with the cerebellum coordinating movement, indicating a cerebellar lesion.

Note- ataxia can occour in MS

34
Q

Card 3 - Multiple sclerosis:
- 2 Investigations
- long term management
- relapse management

A

Symptomatic treatments include:

Exercise to maintain activity and strength
Fatigue may be managed with amantadine, modafinil or SSRIs
Neuropathic pain may be managed with medication (e.g., amitriptyline or gabapentin)
Depression may be managed with antidepressants, such as SSRIs
Urge incontinence may be managed with antimuscarinic medications (e.g., solifenacin)
Spasticity may be managed with baclofen or gabapentin
Oscillopsia may be managed with gabapentin or memantine
MS is a clinical diagnosis based on symptoms that suggest legions that change location overtime.

Ix:
- MRI scans demonstrate typical legions
- Lumbar Puncture can detect oliglonal bands in the CSF

Management:
- Main treatment - disease modifying therapies aim to induce long term-remission.

Relapses are managed with steroids (IV if severe).

Symptomatic treatments include:

Exercise to maintain activity and strength
Fatigue may be managed with amantadine, modafinil or SSRIs
Neuropathic pain may be managed with medication (e.g., amitriptyline or gabapentin)
Depression may be managed with antidepressants, such as SSRIs
Urge incontinence may be managed with antimuscarinic medications (e.g., solifenacin)
Spasticity may be managed with baclofen or gabapentin
Oscillopsia (vision problem where surrounding appear to be moving) may be managed with gabapentin or memantine

35
Q

Pupil Disorders CARD
- card including Third nerve palsy and horner’s syndrome

A
36
Q

Dyarthria vs Dysphagia vs Dysphasia

A

Dysarthria - Slurred Speech (caused by motor weakness)

Dysphagia - difficulty swallowing (phago - eating)

Dysphasia - difficulty with language. Expressive or receptive - damage to language centres in the brain

37
Q

Lower vs Upper Motor Neurone signs

A

The upper motor neurons originate in the cerebral cortex and travel down to the brain stem or spinal cord, while the lower motor neurons travel from the spinal cord and innervate muscles and glands throughout the body. Basically those in the spinal cord are upper motor neurones (part of the CNS).

Signs of lower motor neurone disease:
Weakness - both
Reduced tone
Reduced reflexes
Fasciculations (twitches in the muscles) - spotnaneous firing of remaining muscle units.
Muscle wasting - rember as no tone

Signs of upper motor neurone disease:
Weakness- both
Increased tone or spasticity
Increased reflexes
Upgoing plantar reflex

There is weakness but no wasting initially (i assume because of the increased tone) and no fasciculations.

Just remeber as everything going down vs up - - tone, reflexes, muscle bulk. Fascilations are the exception

Both upper and lower cause muscle weakness but UMP are typically large groups of muscles in a pyramidal distribution (distally - face, arms and legs) whereas LMN tend to be more specific muscle groups.

38
Q

Motor Neurone Disease:
- what is motor neurone disease?
- what types of motor neurones are affected?
- Most common Types?
- Presentation?
- UMN vs LMN signs

A

Motor neurone disease is a term that encompasses a variety of specific diseases affecting the motor nerves.
Motor neurone disease is a progressive, eventually fatal condition where the motor neurones stop functioning.

Pathophysiology: Motor neurone disease involves a progressive degeneration of both the upper and lower motor neurones. The sensory neurones are spared.**There is no effect on the sensory neurones. Sensory symptoms suggest an alternate diagnosis.
**

Types:
Amyotrophic lateral sclerosis (ALS) is the most common and well-known type of motor neurone disease. Stephen Hawking had amyotrophic lateral sclerosis.

Progressive bulbar palsy is the second most common form of motor neurone disease. It primarily affects the muscles of talking and swallowing (the bulbar muscles).

Other types to be aware of are progressive muscular atrophy and primary lateral sclerosis.

The cause is not known but it there are links with genetics, smoking and exposure to heavy metals and pesticides.

///

The typical patient is a late middle-aged (e.g., 60) man, possibly with an affected relative.

There is an insidious, progressive weakness of the muscles throughout the body, affecting the limbs, trunk, face and speech. The weakness is often first noticed in the upper limbs. There may be increased fatigue when exercising. They may complain of clumsiness, dropping things more often or tripping over. They can develop slurred speech (dysarthria).

Signs of lower motor neurone disease:

Muscle wasting
Reduced tone
Fasciculations (twitches in the muscles)
Reduced reflexes

Signs of upper motor neurone disease:

Increased tone or spasticity
Brisk reflexes
Upgoing plantar reflex

39
Q

Motor Neurone Disease 2:
Diagnosis (3 Ix to support diagnosis) and Management (2 key things) ?

A

Diagnosis:
- clinical, after excluding other conditions (must be certain)

  • EMG and nerve conduction studies are useful (wouldnt bother learning findings but know that they are abnormal)
  • MRI Brain and spine - usually normal

Management:
- no effective treatments for halting or reversing the disease
- Riluzole - can slow progression and extend survival by several months in ALS
- Non-invasive ventilation- when respiratory muscles weaken

Symptomatic measures - dont focus learning on these.
- baclofen for muscle spasticity
- anti-muscurinics for excessive saliva
- Benzodiazepines may help breathlessness worsened by anxiety

Prognosis - patients tend to die of repsiratory failure or pneumonia. Most people die within 5 years of diagnosis.

40
Q

What muscles control pupil size and what nerves are involved? Fairly easy but key

cocain vs opiates

A

The circular muscles in the iris are responsible for pupil constriction. They are stimulated by the parasympathetic nervous system using acetylcholine as a neurotransmitter. The parasympathetic fibres travel along the oculomotor nerve (cranial nerve III).

The dilator muscles in the iris are responsible for pupil dilation. They are arranged like spokes on a bicycle wheel, travelling straight from the inside to the outside of the iris. They are stimulated by the sympathetic nervous system, using adrenalin as a neurotransmitter?

Remeber in flight or flight need to let light in to see the lion clearly

Cocain is a stimulant so like adrenline and causes dilated pupils, opiates cause constricted pupils - parasympathethetic

41
Q

What does a third nerve palsy result in - triad?

Causes of a fixed dilated pupil?

What is horner’s syndrome - triad?

Cuases of horner’s syndrome?

A

A palsy in the third cranial nerve (the oculomotor nerve) causes:
- Ptosis (drooping upper eyelid)
- Dilated non-reactive pupil
- Divergent strabismus (squint) in the affected eye, with a “down and out” position of the affected eye

Explaination:
Divergent strabismus (squint) in the affected eye, with a “down and out” position of the affected eye - most of the extraocular muscles stop working, and the lateral rectus and superior oblique (which are still working) pull the eye downward and outward

Ptosis (drooping upper eyelid) - The oculomotor nerve also supplies the levator palpebrae superioris, which is responsible for lifting the upper eyelid. Therefore, a third nerve palsy causes ptosis (drooping of the upper eyelid).

Dilated non-reactive pupil -The oculomotor nerve carries parasympathetic fibres that innervate the circular muscles of the iris. Therefore, third nerve palsy causes a dilated, non-reactive pupil.

Third nerve palsy’s can affect the pupil (compression of the nerve and parasympathetic fibres) or be pupil sparing depending on the cause

Compression affecting pupil:
- tumour
- trauma
- raised ICP
- PCA aneurism
- cavernous sinus thromobosis
- note - the occulomotor nerve travels through the cavernous sinus and close to the posterior communicating artery

///
Horner syndrome involves a triad of:

Ptosis
Miosis
Anhidrosis (loss of sweating)

Horner syndrome is caused by damage to the sympathetic nervous system supplying the face.

The sympathetic nerves arise from the spinal cord in the chest. These are pre-ganglionic nerves. They enter the sympathetic ganglion at the base of the neck and exit as post-ganglionic nerves. The post-ganglionic nerves travel to the head alongside the internal carotid artery. The location of the Horner syndrome can be determined by the anhidrosis (loss of sweating). Central lesions (occurring before the nerves exit the spinal cord) cause anhidrosis of the arm, trunk and face. Pre-ganglionic lesions cause anhidrosis of the face. Post-ganglionic lesions do not cause anhidrosis.

Central causes - stroke, brian tumours, MS
Pre-ganglionic - pancoast tumour, trauma/thyroidectomy
Post-ganglionic - carotid issues (aneurism, disection), cluster headaches

Cocaine eye drops can be used to test for Horner syndrome. It acts on the eye to stop noradrenalin re-uptake at the neuromuscular junction. This causes a normal eye to dilate as noradrenalin stimulates the dilator muscles of the iris. In Horner syndrome, the nerves are not releasing noradrenalin, so blocking re-uptake makes no difference, and there is no pupil reaction.

42
Q
A
43
Q

Headache Red flag symptoms and what they mean

A

Red Flags

Key red flags associated with a headache, indicating a possible serious underly cause, include:

  • Fever, photophobia or neck stiffness (meningitis, encephalitis or brain abscess) - fever and neckstiffness are specific to infection, photophobia also occours in migraine
  • New neurological symptoms (haemorrhage or tumours)
  • Visual disturbance (giant cell arteritis, glaucoma or tumours)
  • Sudden-onset occipital headache (subarachnoid haemorrhage)
  • Worse on coughing or straining (raised intracranial pressure)
  • Postural, worse on standing, lying or bending over (raised intracranial pressure)
  • Vomiting (raised intracranial pressure or carbon monoxide poisoning)
  • History of trauma (intracranial haemorrhage)
  • History of cancer (brain metastasis)
  • Pregnancy (pre-eclampsia)

Fundoscopy for papilloedema is an important. Papilloedema suggests raised intracranial pressure, which may be due to a brain tumour, benign intracranial hypertension or an intracranial bleed.

44
Q

Tension headaches:
- what are they/symptoms?
- management?
- management of chronic tension headaches?
- what is a secondary headache?
- patient with chronic pain might develop headaches due to what?

A

Tension headaches are very common. They typically cause a mild ache or pressure in a band-like pattern around the head. They develop and resolve gradually and do not produce visual changes.

Tension headaches may be associated with:

Stress
Depression
Alcohol
Skipping meals
Dehydration

Management is with:

Reassurance
Simple analgesia (e.g., ibuprofen or paracetamol)

Amitriptyline is generally first-line for chronic or frequent tension headaches.

//

Secondary headaches give a similar presentation to a tension headache but with a clear cause, such as:

Infections (e.g., viral upper respiratory tract infection)
Obstructive sleep apnoea
Pre-eclampsia
Head injury
Carbon monoxide poisoning

Basically they are just a symtoms of something wider going on, not the main symptom like in dehydration.

//

Medication-overuse headache (also called analgesic headache) is a headache caused by frequent analgesia use. It gives similar non-specific features to a tension headache. Withdrawal of the analgesia is important in treating the headache, although this can be challenging in patients with long-term pain.

//

45
Q

Give a cause of a frontal headache with tenderness on palpation?
Management?

A

Sinusitis refers to inflammation of the paranasal sinuses in the face. It typically causes pain and pressure following a recent viral upper respiratory tract infection. There may be tenderness and swelling on palpation of the affected areas. Most cases are caused by a viral infection and resolve within 2-3 weeks.

Prolonged cases (over 10 days) (most are relf resolving) may be treated with a steroid nasal spray or antibiotics (phenoxymethylpenicillin first-line).

46
Q

How do hormonal headaches present and when?

Management?

A

Hormonal headaches are related to low oestrogen. They have similar features to migraines, with a unilateral, pulsatile headache associated with nausea. They are sometimes called menstrual migraines. They may occur:

Two days before and the first three days of the menstrual period
In the perimenopausal period
Early pregnancy (headaches in the second half of pregnancy should prompt investigations for pre-eclampsia)

Triptans and NSAIDs (e.g., mefenamic acid) are treatment options.

47
Q

A Patient gets an eletric shock-like pain on one side of the face when talking, eating or going outside into cold air. The pain can last for a few hours.

What is the diagnosis?

What is the management?

A

Trigeminal neuralgia causes intense facial pain in the distribution of the trigeminal nerve, which has three branches:

Ophthalmic (V1)
Maxillary (V2)
Mandibular (V3)

Trigeminal neuralgia can affect any combination of the branches. Over 90% of cases are unilateral. It is more common in patients with multiple sclerosis.

The pain comes on suddenly and can last seconds to hours. It may be described as an electricity-like, shooting, stabbing or burning pain. It may be triggered by touch, taking, eating, shaving or cold. Attacks may worsen over time.

NICE CKS (updated 2022) recommend carbamazepine as first-line for trigeminal neuralgia. Various surgical interventions are possible where the symptoms persist.

48
Q

What is the definition of a migraine - 4 features?

Other assocaited features of the headaches?

4 types of migraine?

what is aura?

A

A headache lasting 4-72 hours with at least two of the following features:
- Usually unilateral but can be bilateral
- Moderate-severe intensity
- Pounding or throbbing in nature
- Avoidance of usual activities - aggrevated by walking aorund

The headache is usually associated with
- Nausea and vomiting
- photophobia
- phonophobia
- otophobia
- may or may not be associated with Aura (visual changes)

Migraine can be categorised into four main types:
Migraine without aura
Migraine with aura
Silent migraine (migraine with aura but without a headache)
Hemiplegic migraine - unilateral limb weakness, ataxia or loss of conciousness. Important to rule out stroke which has sudden rather than gradual onset hemiplegia.

Aura?

Aura can affect vision, sensation or language. Visual symptoms are the most common. These may be:

Sparks in the vision
Blurred vision
Lines across the vision
Loss of visual fields (e.g., scotoma)

Sensation changes may include tingling or numbness. Language symptoms include dysphasia (difficulty speaking).

Migraine’s can be triggered by:
- Stress
- Bright lights
- Strong smells
- Certain foods (e.g., chocolate, cheese and caffeine)
- Dehydration
- Menstruation
- Disrupted sleep
- Trauma

So basically i think that the trigger cant help you diagnose migrane vs headahce, its more about the features, and intensity of the headache.

49
Q

KEY - Migraines Card 2 - Management:

  1. If remeber nothing else focus on 4 things for acte management and 3 prophalactic options for chronic migraines?

Questions:
- acute management of migraines
- how do triptans work, and what are the contra-indications?
- prophalaxis of migraines - 3 usual medications? more specialst options?, non-medical therapy?
- prophalaxis for menstrual migraines?
- risk of migranes and being on the CCP?

A

Patients may develop strategies for managing symptoms, often retreating to a dark, quiet room and sleeping.

Medical options for an acute attack are:

NSAIDs (e.g., ibuprofen or naproxen)
Paracetamol
Triptans (e.g., sumatriptan)
Antiemetics if vomiting occurs (e.g., metoclopramide or prochlorperazine)

Opiates are not used to treat migraines and may make the condition worse.

///

Triptans are used to abort migraines when they start to develop. They are 5-HT receptor agonists (they bind to and stimulate serotonin receptors), specifically 5-HT1B and 5-HT1D. They have various mechanisms of action, including:

Cranial vasoconstriction
Inhibiting the transmission of pain signals
Inhibiting the release of inflammatory neuropeptides

The main contraindications relate to risks associated with vasoconstriction, for example, hypertension, coronary artery disease or previous stroke, TIA or myocardial infarction.

///

Prophalaxis

The usual prophylactic medications to reduce the frequency and severity of attacks are:

Propranolol (a non-selective beta blocker)
Amitriptyline (a tricyclic antidepressant)
Topiramate (antiepileptic- teratogenic and very effective contraception is needed)

More specialist options include:

Pizotifen
Candesartan
Sodium valproate
Monoclonal antibodies (e.g., erenumab and fremanezumab)

Other options mentioned in the NICE clinical knowledge summaries (updated 2022) include:

Cognitive behavioural therapy
Mindfulness and meditation
Acupuncture
Vitamin B2 (riboflavin)

///

Prophylactic triptans (e.g., frovatriptan or zolmitriptan) are an option for menstrual migraines. Symptoms tend to occur two days before until three days after the start of menstruation. Regular triptans may be taken during this time.

Migraines tend to become less frequent and severe or stop altogether with time, particularly after menopause.

Migraines are associated with a slightly increased risk of stroke, particularly when associated with aura. The risk of stroke is further increased with the combined contraceptive pill, making them a contraindication to the combined pill.

50
Q

Cluster Headaches:
- describe the symptoms/features of the headache?
- Why are they called cluster headaches?
- treatment of acute attacks -2?
- first line for prophalaxis? If you can, what are the other options?

A

Cluster headaches are severe and unbearable unilateral headaches, usually centred around the eye.

Cluster headaches cause severe pain. They are sometimes called “suicide headaches” due to their severity.

Associated symptoms are typically unilateral:

Red, swollen and watering eye
Pupil constriction (miosis)
Eyelid drooping (ptosis)
Nasal discharge
Facial sweating

They are called cluster headaches as they come in clusters of attacks and then disappear for extended periods. For example, a patient may suffer 3-4 episodes a day for weeks or months, followed by a pain-free period lasting several years. Attacks last between 15 minutes and 3 hours.

A typical patient is a 30-50 year old male smoker. They may have triggers, such as alcohol, strong smells or exercise.

////
Management

Treatment options during acute attacks are:
- Triptans (e.g., subcutaneous or intranasal sumatriptan)
- High-flow 100% oxygen (may be kept at home)

Verapamil is the first line for prophylaxis (to prevent attacks).

Other options for prophylaxis are:

Occipital nerve block
Prednisolone (e.g., a short course to break the cycle during clusters)
Lithium

51
Q

What is Neurofibromatosis?

Pathophysiology?

Features?

What are two unique complications of neurofibromatosis?

Neurofibromatosis 2?

A

Neurofibromatosis is a genetic condition that causes nerve tumours (neuromas) to develop throughout the nervous system. These tumours are benign but can cause neurological and structural problems.

Neurofibromatosis type 1 is more common than neurofibromatosis type 2. This section focuses mainly on type 1.

Pathophysiology:
The neurofibromatosis type 1 gene is found on chromosome 17. It codes for a protein called neurofibromin, which is a tumour suppressor protein. Mutations in this gene are inherited in an autosomal dominant pattern.

Features:
The diagnostic criteria for neurofibromatosis type 1 are based on the features, remembered with the “CRABBING” mnemonic:

Note- clinical diagnosis but genetic testing can help confirm

C – Café-au-lait spots (more than 15mm diameter is significant in adults)
R – Relative with NF1
A – Axillary or inguinal freckling
BB – Bony dysplasia, such as Bowing of a long bone or sphenoid wing dysplasia
I – Iris hamartomas (Lisch nodules), which are yellow-brown spots on the iris
N – Neurofibromas
G – Glioma of the optic pathway

Neurofibromas are peripheral nerve tumours that may be seen on the skin. They are skin-coloured, raised nodules or papules with a smooth, regular surface. A single skin neurofibroma without other features does not indicate neurofibromatosis. Two or more are significant. A plexiform neurofibroma is a larger, irregular, complex neurofibroma containing multiple cell types. A single plexiform neurofibroma is significant.

Management - There is no treatment for hte udnerlying disease progress. Just management of complications:
Migraines
Epilepsy
Renal artery stenosis, causing hypertension
Learning disability
Behavioural problems (e.g., ADHD)
Scoliosis of the spine
Vision loss (secondary to optic nerve gliomas)
**Malignant peripheral nerve sheath tumours
**
**Gastrointestinal stromal tumour (a type of sarcoma)
**Brain tumours
Spinal cord tumours with associated neurology (e.g., paraplegia)
Increased risk of cancer (e.g., breast cancer and leukaemia)

TOM TIP: The two unique complications worth remembering for NF1 are malignant peripheral nerve sheath (MPNST) and gastrointestinal stromal tumours (GIST).

52
Q

What condition is associated with bilateral accoustic neuromas (vestibular schwannommas)?

A

Neurofibromatosis type 2

Explaination:
The neurofibromatosis type 2 gene is found on chromosome 22. It codes for a protein called merlin, a tumour suppressor protein important in Schwann cells. Schwann cells provide the myelin sheath that surrounds neurones of the peripheral nervous system. Mutations in this gene lead to schwannomas (benign tumours of the Schwann cells). Inheritance is also autosomal dominant.

Neurofibromatosis type 2 is particularly associated with acoustic neuromas, which are tumours of the auditory nerve that innervates the inner ear.

Surgery can be used to resect the tumours, although there is a risk of permanent nerve damage.

TOM TIP: An exam patient with bilateral acoustic neuromas almost certainly has neurofibromatosis type 2.

53
Q

Key:

Stenosis most commonly occours in which section of the spine?

Key - spinal anatomy
What are the three types of spinal stenosis?
Antomy basics - What are spinal nerve roots? vs spinal nerves?

A

Spinal stenosis refers to the narrowing of part of the spinal canal, resulting in compression of the spinal cord or nerve roots. This usually affects the cervical or lumbar spine. This section focuses on lumbar spinal stenosis, which is the most common type.

There are three types:
Central stenosis – narrowing of the central spinal canal. The spinal canal is where the spinal cord and cauda equina (nerve roots below L1/L2) are located.

Lateral stenosis – narrowing of the nerve root canals. The lateral recess is part of the spinal canal, located just before the nerve root exits the spine through the intervertebral foramen. It is essentially a space in the spinal canal that the ventral or dorsal nerve root passes through before merging.

Foramina stenosis – narrowing of the intervertebral foramina. The intervetrbal formania is the opening between two adjacent vetebrae that allows the spinal nerve to exit the spinal canal to the periphery.

**Note- the spinal nerve roots are bundles of nerve fibres that exit the spinal cord. The ventral roots carry motor fibres and the dorsal roots cary sensory fibres. The dorsal and ventral nerve roots merge at the intervertebral foramen to form one spinal nerve. **

Just to clarify - Path of the Nerve:

1.	Nerve Root: The nerve root travels from the spinal cord through the lateral recess and enters the intervertebral foramen.
2.	Spinal Nerve: Inside the foramen, the dorsal and ventral roots join to form a spinal nerve, which then exits the foramen to innervate various parts of the body.

///

54
Q

Spinal Stenosis - Card 2

Causes of spinal stenosis?

Presentation of central stenosis?

Presentaiton of lateral and formina stenosis?

A

NOTE - Remember there are three types of spinal stenosis - central, lateral and formina stenosis

Causes

Several conditions can cause the spinal canal to narrow, including:

Degenerative changes, including facet joint changes, disc disease and bone spurs (osteophytes) - osteophyte formation most common cause.
Herniated discs
Thickening of the ligamenta flava or posterior longitudinal ligament
^^^ learn these mainly
Congenital spinal stenosis
Spinal fractures
Spondylolisthesis (anterior displacement of a vertebra out of line with the one below)
Tumours

///
Presentation:
Gradual onset vs cauda equina syndrome
Severity of symptoms depends on the degree of compression. Mild symptoms with mild compression, severe compression can lead to cauda equina syndrome

Central stenosis of the lumbar spine - presents with Intermittent neurogenic claudication/ also known as pseudoclaudication Typical symptoms are:

Lower back pain
Buttock and leg pain
Leg weakness

The symptoms are absent at rest and when seated but occur with standing and walking. Bending forward (flexing the spine) expands the spinal canal and improves symptoms. Standing straight (extending the spine) narrows the canal and worsens the symptoms.

TOM TIP: The important thing for your exams is to spot the typical symptoms of intermittent neurogenic claudication. At first glance, they are similar to peripheral arterial disease. The exam question might specify that the peripheral pulses or the ankle-brachial pressure index (ABPI) are normal, in which case the diagnosis is more likely to be spinal stenosis. Additionally, patients with spinal stenosis are more likely to struggle with back pain, whereas back pain is not a feature of peripheral arterial disease.

Presentation of Lateral and formina stenosis - Radiculopathy, for example sciatica if involving sciatic nerve roots

The term radiculopathy refers to compression of the nerve roots as they exit the spinal cord and spinal column, leading to motor (weakness) and sensory symptoms (numbness and paratehsia), and radicular pain.

So basically the symptoms are really similar…

55
Q

From supervisor - radiculopathy vs myelopathy vs spinal stenosis

A

Radiculopathy - compression of a nerve root exiting the spine (often cervical or lumbar). Usually due to a protruding intervertebral disc but any abnormal growth can compress.

Symptoms:
(1) pain (could be neck or back) but also in the distribution of that nerve root (dermatomes)
(2) weakness in that distribution (myotomes)

Signs:
(1) Weakness in that myotome
(2) Reflexes absent/reduced (e.g. triceps in C7)
(3) Sensory impairment in that dermatome

Treatment:
1) Usually conservative with physio
2) Can have nerve root blocks or surgical decompression

//

Myelopathy - this refers to a disorder of the spinal cord. In the surgical context it means compression but there are various medical causes e.g. B12 deficiency. Surgical causes are often disc protrusion indenting the cord and disrupting blood flow. This is UMN as it’s the spinal cord. Often cervical (remember spinal cord ends in high lumbar region).

Symptoms:
1) Variable depending on which part of the cord is affected. Can affect everything below cervical level, too. Could present as hand clumsiness, leg weakness, bowel/bladder dysfunction. Usually gradually worse over time.

Signs:
1) UMN - could have stiff legs with pyramidal weakness and brisk reflexes
2) May have a sensory level.

Spinal Stenosis:

Often lumbar. Narrowing of the canal. There’s the nerve roots at this point. Symptoms are dynamic e.g. leg weakness and pain that occurs after walking downhill (where canal is more narrow).

Spine narrows predominately through athritic changes.

56
Q

Spinal Stenosis Card 3

Key Ix?

Management?

A

Ix - MRI is the primary imaging investigation for diagnosing spinal stenosis.

Management:
- analgesia
- physiotherapy
- exercize and weight loss
- Spinal Decompression surgery

Laminectomy refers to the removal of part the lamina from the affected vertebra - The laminae are the bony parts that form the posterior part of the vertebral foramen (forming the spinal canal) and attaches to the spinous process.

57
Q

What is cauda equina syndrome?

At what spinal level does the spinal cord terminate?

What is the most common cause of cauda equina syndrome?

Red flags suggesting cauda equina?

Management - Ix and Rx?

A

Cauda equina syndrome is a surgical emergency where the nerve roots of the cauda equina at the bottom of the spine are compressed. It requires emergency decompression surgery to prevent permanent neurological dysfunction. However, even with immediate decompression, patients may still not regain full function.

The cauda equina (translated as “horse’s tail”) is a collection of nerve roots that travel through the spinal canal after the spinal cord terminates around L2/L3. The spinal cord tapers down at the end in a section called the conus medullaris. The nerve roots exit either side of the spinal column at their vertebral level (L3, L4, L5, S1, S2, S3, S4, S5 and Co).

The nerves of the cauda equina supply:

Sensation to the lower limbs, perineum, bladder and rectum
Motor innervation to the lower limbs and the anal and urethral sphincters
Parasympathetic innervation of the bladder and rectum

In cauda equina syndrome, the nerves of the cauda equina are compressed. There are several possible causes of compression, including:

Herniated disc (the most common cause)
Tumours, particularly metastasis
Spondylolisthesis (anterior displacement of a vertebra out of line with the one below)
Abscess (infection)
Trauma

///

The key red flags to look out for are:

Saddle anaesthesia (loss of sensation in the perineum – around the genitals and anus)
Loss of sensation in the bladder and rectum (not knowing when they are full)
Urinary retention or incontinence
Faecal incontinence
Bilateral sciatica
Bilateral or severe motor weakness in the legs
Reduced anal tone on PR examination

Remeber radiculopathy is unilateral, and lumbar stenosis is intermittent symptoms not constant. But i think central stenosis can cause bilateral symptoms?

///

Management

Cauda equina is a neurosurgical emergency. It requires:

Immediate hospital admission
Emergency MRI scan to confirm or exclude cauda equina syndrome
Neurosurgical input to consider lumbar decompression surgery

Surgery should be performed as soon as possible to increase the chances of regaining function. Even with early surgery, patients can be left with bladder, bowel or sexual dysfunction. Leg weakness and sensory impairment can also persist.

///

TOM TIP: Cauda equina presents with lower motor neuron signs (reduced tone and reduced reflexes). The nerves being compressed are lower motor neurons that have already exited the spinal cord. When the spinal cord is being compressed higher up by metastatic spinal cord compression (different but also an emergency), upper motor neuron signs (increased tone, brisk reflexes and upping plantar responses) will be seen.

58
Q

What is sciatica?
What nerve roots form the sciatic nerve and which two nerves does the sciatic nerve branch into?
Causes of sciatica?
management?

A

Sciatica refers to lumbar radiculopathy and describes UNILATERAL radiating leg pain caused by inflammation or compression of the lumbosacral nerve roots (L4–S1) forming the sciatic nerve.

The spinal nerves **L4 – S3 come together to form the sciatic nerve. **The sciatic nerve exits the posterior part of the pelvis through the greater sciatic foramen, in the buttock area on either side. It travels down the back of the leg. **At the knee, it divides into the tibial nerve and the common peroneal nerve. **

The sciatic nerve supplies sensation to the lateral lower leg and the foot. It supplies motor function to the posterior thigh, lower leg and foot.

Sciatica causes unilateral pain from the buttock radiating down the back of the thigh to below the knee or feet. It might be described as an “electric” or “shooting” pain. Other symptoms are paraesthesia (pins and needles), numbness and motor weakness. Reflexes may be affected depending on the affected nerve root.

A positive result in a straight leg raise test — which means with the person lying supine, the hip is flexed gradually with the knee extended. Pain reproduced below 60 degrees of hip flexion on the ipsilateral side indicates a positive test.

The main causes of sciatica are lumbosacral nerve root compression by:

Herniated disc
Spondylolisthesis (anterior displacement of a vertebra out of line with the one below)
Spinal stenosis

Bilateral sciatica is a red flag for cauda equina syndrome requiring emergency MRI

Note - Cervical spondylosis (degenerative changes to the vertebrae) and can result in neck pain and headaches.

//

Management
The NICE clinical knowledge summaries (updated 2020) state not to use medications such as gabapentin, pregabalin, diazepam or oral corticosteroids for sciatica. They state not to use opioids for chronic sciatica.

Sciatica symptoms usually settle within a few weeks to a few months, but in some people they may persist for longer and recurrence is common.

They suggest considering a neuropathic medication if symptoms are persisting or worsening at follow up, but not gabapentin or pregabalin, leaving at the main choices of:
**
Amitriptyline
Duloxetine
**

Specialist management options for chronic sciatica include:

Epidural corticosteroid injections
Local anaesthetic injections
Radiofrequency denervation
Spinal decompression
referal for physiotherapy

59
Q

Fascial nerve palsy:
- how does it present?
- how do you distinguish UMN from LMN facial nerve palsy?
- funtions of the facial nerve? 3
- Bells palsy - cause, and management?
- fascial nerve palsy with a painful vesicular rash around the ear - cause and management?

A

Facial nerve palsy refers to isolated dysfunction of the facial nerve and presents with unilateral facial weakness: drooping of the eyelid, loss of the nasolabial fold.

///

Facial Nerve palsy - LMN forehead is affected.
It is essential to distinguish between upper motor neurone and lower motor neurone facial nerve palsy. Patients with new-onset upper motor neurone facial nerve palsy need immediate management as a possible stroke. In contrast, patients with lower motor neurone facial nerve palsy can be managed less urgently.

Each side of the forehead has upper motor neurone innervation by both sides of the brain. However, each side of the forehead only has lower motor neurone innervation from one side of the brain.

The forehead will be spared in an upper motor neurone lesion so the patient can move their forehead on the affected side. In a lower motor neurone lesion, the forehead is not spared, and the patient cannot move their forehead on the affected side.

Biggest cause of unilateral UMN (so forehead sparing) legion causing facial weakness is stroke, bilateral is rare and occours in peudobulbar palsy and motor neurone disease.

///

Motor funtions to the muscles of fascial expression, stapedius in the inner ear and some of the throat muscles

Sensory - taste for the anterioir 2/3 of the tongue

Parasympathetic - lacrimal (tears) and submandibular and sublingual salviary glands.

///
Bell’s palsy is a relatively common condition. It is idiopathic, meaning there is no apparent cause. It presents with a unilateral lower motor neurone facial nerve palsy. Most patients fully recover over several weeks, but recovery may take up to 12 months. A third are left with some residual weakness.

If patients present within 72 hours of developing symptoms, NICE clinical knowledge summaries (updated 2023) recommend considering prednisolone as treatment, either:

50mg for 10 days
60mg for 5 days followed by a 5-day reducing regime, dropping the dose by 10mg per day

Patients also require lubricating eye drops to prevent the eye from drying out and being damaged. If they develop pain in the eye, they need an ophthalmology review for exposure keratopathy. The eye can be taped closed at night.
///
Ramsay-Hunt syndrome is caused by the varicella zoster virus (VZV). It presents as a unilateral lower motor neurone facial nerve palsy. Patients stereotypically have a painful and tender vesicular rash in the ear canal, pinna and around the ear on the affected side. This rash can extend to the anterior two-thirds of the tongue and hard palate.

Treatment is with aciclovir and prednisolone. Patients also require lubricating eye drops.

TOM TIP: Ramsay-Hunt syndrome is popular in MCQ exams. Look out for the patient with facial nerve palsy and a vesicular rash around their ear.

Note - remeber there are other causes of a fascial nerve palsy but these are the most common. MS, parotid tumour…

60
Q

Presentation of brain tumours - 2 categories of symptoms? Give examples

Finding on examination?

A

Brain tumours may be asymptomatic, particularly when they are small. As they grow, they present with progressive focal neurological symptoms depending on the location of the lesion. TOM TIP: A common exam scenario is an unusual change in personality and behaviour, which indicates a frontal lobe tumour. The frontal lobe is responsible for personality and higher-level decision-making.

Brain tumours often present with symptoms and signs of raised intracranial pressure (intracranial hypertension). A growing tumour takes up room within the skull, leaving less space for the other contents (such as the cerebrospinal fluid), causing a rise in the pressure within the intracranial space.

In patients presenting with headaches, the concerning features that may indicate intracranial hypertension include:

Constant headache
Nocturnal (occurring at night)
Worse on waking
Worse on coughing, straining or bending forward
Vomiting
Papilloedema on fundoscopy

There are additional symptoms of raised ICP not relating to headache- seizsures, visual field defects…

Papilloedema is a crucial fundoscopy finding in patients with increased intracranial pressure. Papilloedema describes swelling of the optic disc secondary to raised intracranial pressure. Papill- refers to a small, rounded, raised area (the optic disc) and -oedema refers to the swelling.

The sheath around the optic nerve is connected with the subarachnoid space. The raised cerebrospinal fluid (CSF) pressure flows into the optic nerve sheath, increasing the pressure around the optic nerve behind the optic disc causing the optic disc to bulge forward.

61
Q

Types of brain tumour:

gliomas:
- what are glial cells
- Name of the most malignant gliomas
- name 3 types of glioma

Meningiomas - how do they cause symptoms?

Secondary metastasis - what cancers commonly metastize to the brain

Pituitary adenomas - symptoms 5 and management 4

Acoustic neuroma - what is it? symptoms? management?

A

Gliomas are tumours of the glial cells in the brain or spinal cord. Glial cells surround and support the neurones. Glial cells include astrocytes, oligodendrocytes and ependymal cells.

Gliomas are graded from 1 to 4. Grade 1 is the most benign (possibly curable with surgery), and grade 4 is the most malignant (e.g., glioblastoma multiforme). The main three types (roughly from most to least malignant) are:

Astrocytoma (the most common and aggressive form is glioblastoma)
Oligodendroglioma
Ependymoma

//

Meningiomas

Meningiomas are tumours growing from the cells of the meninges. They are usually benign. However, they take up space, and this “mass effect” can lead to raised intracranial pressure and neurological symptoms.

//

The cancers that most often spread to the brain are:

Lung
Breast
Renal cell carcinoma
Melanoma

Me - whilst glioblastoma is the most common primary brain cancer, secoondary brain cancer is far more common than primary (10:1) so these are significant…

//

Pituitary tumours tend to be benign. If they grow large enough, they can press on the optic chiasm, causing a visual field defect called bitemporal hemianopia, with loss of the outer half of the visual fields in both eyes.

They can cause hormone deficiencies (hypopituitarism) or to release excessive hormones, leading to:

Acromegaly (excessive growth hormone)
Hyperprolactinaemia (excessive prolactin)
Cushing’s disease (excessive ACTH and cortisol)
Thyrotoxicosis (excessive TSH and thyroid hormone)

Pituitary tumours may be managed with:

Trans-sphenoidal surgery (through the nose and sphenoid bone)
Radiotherapy
Bromocriptine to block excess prolactin
Somatostatin analogues (e.g., octreotide) to block excess growth hormone

//
Acoustic Neuroma

Acoustic neuromas are benign tumours of the Schwann cells that surround the auditory nerve (vestibulocochlear nerve) that innervates the inner ear. Schwann cells provide the myelin sheath around neurones of the peripheral nervous system. Acoustic neuromas are also called vestibular schwannomas.

They occur at the cerebellopontine angle and are sometimes called cerebellopontine angle tumours.

They are usually unilateral. Bilateral acoustic neuromas are associated with neurofibromatosis type 2.

The typical patient is a 40-60 year old presenting with a gradual onset of:

Unilateral sensorineural hearing loss (often the first symptom)
Unilateral tinnitus
Dizziness or imbalance
Sensation of fullness in the ear
Facial nerve palsy (if the tumour grows large enough to compress the facial nerve)

Management options include:

Conservative management with monitoring may be used if there are no symptoms or treatment is inappropriate
Surgery to remove the tumour (partial or total removal)
Radiotherapy to reduce the growth

62
Q

Brain cancers 3

Investigations and management for gliomas

A

Investigations:

First line - MRI scan

Gold standard - Biopsy gives the definitive histological diagnosis, usually obtained during surgery to remove the tumour.

Management:
The management of brain tumours depends on the type and grade, guided by the multidisciplinary team. The main options are:

Surgery
Chemotherapy
Radiotherapy
Palliative care

63
Q

What is a mononeuropathy?

Cause?

Most common mononeuropathies? The nerve affected and their symptoms

Carpal Tunnel - Ix and management?

A

Mononeuropathy refers to the damage or dysfunction of a single peripheral nerve.

Fixed mononeuropathies are mainly induced by nerve compression or entrapment of nerves in narrow anatomical spaces.

Most common:
- Cubital tunnel syndrome
- Carpal tunnel syndrome
- peroneal nerve dysnfunction

Carpal tunel syndrome - the median nerve:
- sensation of the first three digets (thumb, forefinger and middle).
- Motor funtion to the three thenar muscles (thumb movements)
- pan and burning sensation

Cubital tunel syndrome - ulnar nerve:
- sensation of the 4th and 5th digits
- weakness/decreased grip strenght
- wasting of the interossi
- worse when the elbow is flexed.

Peroneal nerve dysfunction - peroneal nerve is a branch of the sciatic nerve that provides movement and sensation to the lower leg, foot and toes. Leads to foot drop, wouldnt learn more tbh…

///

Carpal Tunnel
Ix - nerve conductions studies

Management:
Management options for carpal tunnel syndrome are:

Rest and altered activities:
Wrist splints that maintain a neutral position of the wrist can be worn at night (for a minimum of 4 weeks)
Steroid injections
Surgery - flexor retinaculum under local.

64
Q

What is a polyneuropathy?

A

Disorder involving mutliple peripheral nerve fibres, usually occours symetrically and distal nerves are affected first. It usually results in a loss of sensation, parasthesia, neuropathic pain or motor weakness.

Most common causes - diabetes and alcoholism, B12 deficiency

Guillaine Barre is an example of an acute peripheral polyneuropathy..

Differentials:
- radiculopathy - damage to the nerve root causing deficit in a specific dermotomal and myotomal pattern
- mononeuropathy - damage to 1 peripheral nerve
- mononueritis multiplex - multiple peripheral nerves with an asymetrical dystrubution.

Ix - Blood tests for cause
- nerve conduction studies

Rx - based on cause - sugar control for diabetes, vitamin replacement,

65
Q

Define Mononeuritis multiplex

A

MNM is essentially an asymmetrical, asynchronous sensory and motor peripheral neuropathy involving isolated damage to at least two separate limb or cranial nerves. Multiple nerves in random areas of the body can be affected simultaneously or sequentially.

MNM is most commonly caused by vasculitis, which may be either systemic or isolated to the nerves. Other causes include hypersensitivity reactions to drugs or infections, or direct viral or bacterial infection of nerves.

Not a key condition just a wierd and wonderful one to not forget about

66
Q

Vertigo is a descriptive term for a sensation that there is movement between the patient and their environment. Vertigo is often associated with nausea, vomiting, sweating and feeling generally unwell.

How does Meniere’s disease present?

Management?

How does BPPV present?

Diagnosis and managment of BPPV?

A

Remember the typical triad of symptoms in Ménière’s disease, as this is commonly tested in exams:

Hearing loss
Vertigo (episodic and not triggered by movement or posture)
Tinnitus

Vertigo in Ménière’s disease comes in episodes. These last for 20 minutes to several hours before settling. These episodes can come in clusters over several weeks, followed by prolonged periods (often months) without vertigo symptoms.** Vertigo is not triggered by movement or posture.**

These are unilateral, and there may also be a feeling of fullness in the ear. There maybe spantaneous nystagmus in acute attacks also, and people can expereince unexplained falls from loss of balance.

Ménière’s disease is associated with the excessive buildup of endolymph in the labyrinth of the inner ear, causing a higher pressure than normal and disrupting the sensory signals. This increased pressure of the endolymph is called endolymphatic hydrops.

Spontaneous nystagmus may be seen during an acute attack. This is usually in one direction (unidirectional).

Diagnosis is clinical.

For acute attacks, short-term options for managing symptoms include:

Prochlorperazine
Antihistamines (e.g., cyclizine, cinnarizine and promethazine)

Prophylaxis is with:

Betahistine

///
Benign paroxysmal positional vertigo (BPPV) is a common cause of recurrent** episodes of vertigo triggered by head movement. **

Presentation:
A variety of head movements can trigger attacks of vertigo. A common trigger is turning over in bed. Symptoms settle after around 20 – 60 seconds, and patients are asymptomatic between attacks. Often episodes occur over several weeks and then resolve but can reoccur weeks or months later.

BPPV does not cause hearing loss or tinnitus.

Pathophysiology

It is a peripheral cause of vertigo, meaning the problem is located in the inner ear rather than the brain. It is more common in older adults.

BPPV is caused by crystals of calcium carbonate called otoconia that become displaced into the semicircular canals. This occurs most often in the posterior semicircular canal. They may be displaced by a viral infection, head trauma, ageing or without a clear cause. The crystals disrupt the normal flow of endolymph through the canals, confusing the vestibular system. Head movement creates the flow of endolymph in the canals, triggering episodes of vertigo.

Diagnosis - Dix-Hallpike manoeuvre can be used to diagnose BPPV (Dix for Dx – diagnosis). In patients with BPPV, the Dix-Hallpike manoeuvre will trigger rotational nystagmus and symptoms of vertigo. The eye will have rotational beats of nystagmus towards the affected ear (clockwise with left ear and anti-clockwise for right ear BPPV).

Rx - Eppley manoeuvre or Brand-Daroff Exercises

NOTE COMPARE AND CONTRAST BPPV and MENIERE’S - CAN SEE THE BOLD PARTS.

67
Q

Meningitis and Encephalitis from paeds

A
68
Q

Managment of Chronic pain? Including medical?

A

Chronic pain refers to pain that has been present or reoccurs in one or more areas over more than three months. Up to 50% of the adults in the UK are affected by chronic pain.

The NICE guidelines on chronic pain (April 2021) separates chronic pain into:

Chronic primary pain – where no underlying condition can adequately explain the pain
Chronic secondary pain – where an underlying condition can explain the pain (e.g., arthritis)

Biological, psychological and social factors contribute to the persistence and severity of pain. Physical processes that can lead to chronic pain include:

Sensitisation of the primary afferent nociceptors by frequent stimulation
Increased activity of the sympathetic nervous system
Increased muscle contraction in response to pain

MANAGEMENT
Chronic pain is a complex condition that can be challenging to manage. It often fluctuates, with flare-ups, and may get worse over time. Good communication and building a relationship is important. Part of management aims to maintain the quality of life despite the pain.

Options for managing chronic primary pain in the NICE guidelines (2021) include:

Supervised group exercise programs
Acceptance and commitment therapy (ACT)
Cognitive behavioural therapy (CBT)
Acupuncture
Antidepressants (e.g., amitriptyline, duloxetine or an SSRI)
The NICE guidelines (2021) specifically state that for chronic primary pain, patients should not be started on:

Paracetamol
NSAIDs
Opiates
Anti-epileptic drugs (e.g., pregabalin or gabapentin)
In chronic secondary pain, analgesia may be helpful depending on the underlying cause. For example, in patients with osteoarthritis, the pain may be managed with NSAIDs.

TOM TIP: It is worth remembering the NICE guidelines (2021) clearly state to avoid essentially all forms of analgesia in chronic primary pain, except antidepressants.

69
Q

Shingles presentation?

management?

A

Shingles (herpes zoster) is a viral infection of an individual nerve and the skin surface served by the nerve (dermatome). It is caused by the reactivation of the varicella-zoster virus in a spefici dermatome, the same virus that causes chickenpox.

It presents as neuropathic pain (burning, electrical) and other sensory symtpoms in a single dermatomal distribution (usually on the trunk) followed but maculopapular rash in the same dermatome.

Management:
- aciclovir within 72 hours of rash onset
- neuropathic pain management - simple analgesia for mild pain; amitriptyline, duloxetine, gabapentine or prebaline for moderate to severe pain

70
Q

How does giant cell arteritis present?

key complication?

Diagnosis?

A

Giant cell arteritis (GCA) is also known as temporal arteritis. It is a type of systemic vasculitis affecting the medium and large arteries. There is a strong link with polymyalgia rheumatica. It is more common in older white patients.

The key complication of giant cell arteritis is vision loss, which is often irreversible.

//

Presentation:
Unilateral headache is the primary presenting feature, typically severe and around the temple and forehead.

It may be associated with:

Scalp tenderness (e.g., noticed when brushing the hair)
Jaw claudication
Blurred or double vision
Loss of vision if untreated

//

Diagnosis is with raised ESR, temporal artery biospy (showing multinucleated giant cells) and duplex ultrasound shwoing halosign.

Management is with Steroids

71
Q

Meningitis 1:

Causes of bacterial meningitis? And in neonates?

Causes of viral meningitis?

Symptoms?

Investigation on neonates presenting with fever?

Special tests?

A

Meningitis is inflammation of the meninges, usually due to infection. The meninges are the lining of the brain and spinal cord. Cerebrospinal fluid (CSF) is contained within the meninges (in the subarachnoid space).

Bacterial Meningitis

The causes of bacterial meningitis include:

Neisseria meningitidis and
Streptococcus pneumoniae (pneumococcus)

Others:
Haemophilus influenzae
Group B streptococcus (GBS) (particularly in neonates as GBS may colonise the vagina)
Listeria monocytogenes (particularly in neonates)

Note - Meningococcal meningitis is when the bacteria infects the meninges and the cerebrospinal fluid. Meningococcal septicaemia is when the meningococcus bacterial infection is in the bloodstream. Meningococcal septicaemia can cause the classic non-blanching rash

//
The most common causes of viral meningitis are:

Enteroviruses (e.g., coxsackievirus)
Herpes simplex virus (HSV)
Varicella zoster virus (VZV)

Viral PCR testing can be performed on a CSF sample. Aciclovir is used to treat HSV and VZV.

//

Typical symptoms of meningitis are:

Fever
Neck stiffness
Vomiting
Headache
Photophobia
Altered consciousness
Seizures

//

Neonates and babies can present with non-specific signs and symptoms, such as hypotonia, poor feeding, lethargy, hypothermia and a bulging fontanelle. The NICE guidelines on sepsis recommend lumbar puncture as part of the investigations for children with suspected sepsis who are:

Under 1 month, presenting with fever
1 to 3 months and are unwell or have a low or high white blood cell count

//

Kernig’s test involves lying the patient on their back, flexing one hip and knee to 90 degrees and then slowly straightening the knee whilst keeping the hip flexed at 90 degrees. This creates a slight stretch in the meninges. Where there is meningitis, it will produce spinal pain or resistance to movement.

Brudzinski’s test involves lying the patient flat on their back and gently using your hands to lift their head and neck off the bed, flexing their chin to their chest. A positive test, indicating meningitis, is when this causes the patient to flex their hips and knees involuntarily.

72
Q

Meningitis 2

CSF interpretation- describe what occours in the following during bacterial vs viral meningitis:

  • appearence (cloudy/clear)
  • protein
  • glucose
  • White cells - inc cell types
A

TOM TIP: Lumbar puncture interpretation is a common exam question. Think about what will happen with bacteria or viruses living in the CSF rather than rote learning the results. Bacteria swimming in the CSF will release proteins and use up glucose. Viruses may release a small amount of protein and do not use up glucose. The immune system releases more neutrophils with bacteria and lymphocytes with viruses.

Bacterial:
Cloudy
High Protein
Low glucose
Normal
High (neutrophils)

Viral:
Clear
Mildly raised or normal protein
Normal glucose
High Lymphocytes

73
Q

Meningitis 3:

  • investigations in meningitis?
  • Management in primary care?
  • Management of meningitis - antibiotic choice? (inc neonates, and viral)
A

Ix:

Lumbar punture. CSF sent for bacterial culture, viral PCR, cell count, protein and glucose (blood sample also sent at the same time)

Blood cultures

Blood test- meningococcal DNA (faster than blood cultures)

Management:
- primary care - urgent dose of benzylpenicillin (IM or IV) while awaitng tranfer to hospital
- Neonates - cefotaxime + amoxicillin for HSV
- Above 3 months - cefotaxime
- Vancomycin for Penecillin-resistant pneumococcal infection
- Aicclovir for viral meningitis - HSV
- Steroids (e.g., dexamethasone) are also used in bacterial meningitis to reduce the frequency and severity of hearing loss and neurological complications.
- notifiable disease

///
Other bits to be aware of

Significant exposure to meningococcal infection puts contacts at risk. This risk is highest with close prolonged contact within 7 days before the onset of the illness. The usual choice is a single dose of ciprofloxacin given as soon as possible after the diagnosis.

Complications

Hearing loss (a key complication)
Seizures and epilepsy
Cognitive impairment and learning disability
Memory loss
Focal neurological deficits, such as limb weakness or spasticity

74
Q

Encephalitis:

  • most common causes
  • Presentation - you struggle with this
  • Investigations?
  • Management?
A

Encephalitis means inflammation of the brain.

Viral infection is most common:
- Herpes simplex virus is the most common
Others:
- Varicella Zoster
- EBV, Enterovirus (Cocksackie B - the one in meningitis), adenovirus influenza, immunisation viruses (polio measels,

Note similar to meningitis where Cocksackie B, herpes simpex and varicella zoster virus.

Non-infective causes are autoimmune, meaning antibodies are created that target brain tissue.

Presentation:
- fever
- headache
- altered conciousness
- altered congition and confusion
- focal neurological symptoms
- focal seizsures

///

Ix:
- lumbar punture - CSF testing for viral PCR
- CT scan in lumbar puncture is contraindicated (seizsures, GCS below 9)
- MRI scan
- swabs of areas with causative organism
- HIV testing

///
Management:
- aciclovir, started empirically for HSV and VZV
- repeat lumbar puncutre to ensure successful treatment prior to stopping antivirals
- follow up and rehabilitation - complications are common and often recovery is prolongued (basically fatigue and perminant symptoms - perosnality, cognition fatigue, pain…

75
Q

What is Mild cognitive impairment?

Rx?

A

Mild cognitive impairment (MCI) invovles a deficint in cognition (language, reasoning, visual perception) and memory in individuals who maintain the ability to independently perform most activities of daily living - it is milder than dementia (support not required).

Basically it is congitive impairmenet but with preserved function.

MCI can develop for multiple reasons, and individuals living with MCI may go on to develop dementia (I think about half at 5 years); others will not. For neurodegenerative diseases, MCI can be an early stage of the disease continuum including for Alzheimer’s if the hallmark changes in the brain are present.

There are currently no medicines that are licensed to treat MCI or which reduce the chances of developing dementia.

76
Q

Define Demmentia?

What is early onset demmentia?

How does dementia present in the early and later stages?

A

Dementia is a condition that causes progressive and irreversible impairment in memory, cognition, personality and communication. It is particularly associated with older age.

Early-onset dementia refers to when the symptoms start before aged 65.

Early symptoms of dementia include:

Forgetting events
Forgetting names
Difficult remembering words
Repeatedly asking the same questions
Impaired decision making
Reduced flexibility

As the condition progresses, memory and cognitive impairment worsen. Eventually, patients lose the ability to complete self-care tasks such as cooking, cleaning, and dressing themselves.

Features of advanced dementia include:

Inability to speak or understand speech (aphasia)
Swallowing difficulties (dysphagia), which can lead to aspiration and pneumonia
Appetite and weight loss
Incontinence

Behavioural and psychological symptoms of dementia (BPSD) include:

Depression
Anxiety
Agitation
Aggression
Disinhibition (e.g., sexually inappropriate behaviour)
Hallucinations
Delusions
Sleep disturbance

77
Q

What are the 4 main types of demmentia

A

Alzheimers dementia is the most common type of dementia. The underlying pathophysiology involves brain atrophy, amyloid plaques, reduced cholinergic activity and neuroinflammation.

Alzheimer’s and FT - abnormal tau proteins clump together to form neurofibrillary tangles.

Alzhemers- Beta amyloid plaques.

Vascular dementia is the second most common type. It is caused by vascular damage and impaired blood supply to the brain. Risk factors are the same as other cardiovascular diseases (e.g., hypertension, diabetes and smoking).

Dementia with Lewy bodies is a type of dementia associated with features of Parkinsonism. It causes a progressive cognitive decline. There are associated symptoms of visual hallucinations, delusions, REM sleep disorders and fluctuating consciousness. LEARN THESE SPECIFIC 4 THINGS

If motor symptoms predate the cognitive symptoms then it is parkison’s disease demmentia if otherway around then demmentia with lewy bodies
Lewy bodies are build ups of alpha-synuclein that reduced levels of Ach and Dopamine.

Frontotemporal dementia is a rarer type that notably affects people at a younger age (starting aged 40-60). It mainly affects the frontal and temporal lobes. The initial presentation typically involves abnormalities in behaviour, speech and language. It can be familial (inherited). ME- Other cognitive functions (such as memory and perception) may be relatively preserved.

Like alzheimers - abnormal tau proteins clump together to form neurofibrillary tangles.

LEARN THAT IT AFFECTS BEHAVOIR MORE THAN COGNITIVE PERFORMANCE - APATHY, DISINHIBITION…

78
Q

Differential diagnoses for demmentia (cognitive impairement, memory impairement or personality change)

A

Many conditions can cause cognitive impairment, memory impairment or personality changes.

Medications with an anticholinergic effect, particularly:

Anticholinergic urological drugs (e.g., oxybutynin, solifenacin and tolterodine)
Antihistamines (e.g., chlorphenamine and promethazine)
Tricyclic antidepressants (e.g., amitriptyline)

Psychiatric conditions include:

Depression
Psychosis
Delirium (e.g., secondary to infection)

Neurological conditions include:

Brain tumours (particularly affecting the frontal lobes)
Parkinson’s disease
Huntington’s disease
Progressive supranuclear palsy

Endocrine conditions include:

Hypothyroidism
Adrenal insufficiency
Cushing’s syndrome
Hyperparathyroidism (causing hypercalcaemia)

Nutritional deficiencies include:

Vitamin B12 deficiency
Thiamine deficiency (causing Wernicke-Korsakoff syndrome)

79
Q

Modifiable risk factors for dementia

A

Various lifestyle factors have been shown to significantly reduce the risk of developing dementia:

Exercise
Mental stimulation (e.g., a more mentally challenging job)
Maintaining a healthy weight (obesity increases the risk)
Blood pressure control (hypertension increases the risk)
Blood glucose control (diabetes increase the risk)

80
Q

Ix in demmentia

A

Initial blood tests, required to exclude a physical cause before referring to the memory clinic, include:

Full blood count
Urea and electrolytes
Liver function tests
Inflammatory markers (e.g., CRP and ESR)
Thyroid profile
Calcium
HbA1c
B12 and folate

Also Urinalysis and CXR for lung cancer

**Actual investigations:
- The Addenbrooke’s Cognitive Examination-III (ACE-III) is a detailed and comprehensive assessment tool for memory impairment, typically used by specialist memory services. **

DONT NEED TO KNOW THIS -Five domains are tested:

Attention
Memory
Language
Visuospatial function
Verbal fluency

It is scored out of 100 points. Answering perfectly scores 100. A score of 88 or less is typically considered to indicate possible dementia. Lower scores indicate more severe impairment.

Me - CT head can be used for early onset or where a structural pathology is suspected (it often shows hypocampal atrophy). MRI is better for vascular

81
Q

Demmentia Management

  • 4 KEY DRUGS?
  • Rx for BPSD
A

Alzheimer’s disease has drug options to help improve symptoms:

  • FIRST LINE - Acetylcholinesterase inhibitors (e.g., donepezil, rivastigmine or galantamine)
  • SEVERE ALZHEIMER’S - Memantine, which works by blocking N-methyl-D-aspartic acid (NMDA) receptors

BPSD - first treat cause, modifiy environmental factors, music therapy…

Medication options for managing BPSD are only used where necessary and cause significant side effects. Options include:

SSRI antidepressants for depressive symptoms
Antipsychotic drugs (typically risperidone first-line) - behavoiral symptoms
Benzodiazepines (only for crisis management)

ME
Non-pharmalogical
- Cognitive stimulation therapy
- Group reminiscence therapy
- Cognitive rehabilitation or occupational therapy to support functional ability

Vascular demmentia- target the cardiovascular risk factors.